Top Banner
eroberogeday lwm pl:ún briBaØabR&tKNitviTüa nig BaNiC¢kmμ H U N SE N × S T R O N G 3 Problems and Solutions
159

HU NSEN 3 STR O N G...Problems and Solutions . GñkcUlrYmRtYtBinitübec© ... Mathematical Olympiad in China,Mathematical Olympiad Challenges, Mathematical Olympiad Treasures, International

May 28, 2020

Download

Documents

dariahiddleston
Welcome message from author
This document is posted to help you gain knowledge. Please leave a comment to let me know what you think about it! Share it to your friends and learn new things together.
Transcript
Page 1: HU NSEN 3 STR O N G...Problems and Solutions . GñkcUlrYmRtYtBinitübec© ... Mathematical Olympiad in China,Mathematical Olympiad Challenges, Mathematical Olympiad Treasures, International

eroberogeday lwm pl:ún briBaØabR&tKNitviTüa nig BaNiC¢kmμ

H U N S E N×

S T R O N G

3

Problems and Solutions

Page 2: HU NSEN 3 STR O N G...Problems and Solutions . GñkcUlrYmRtYtBinitübec© ... Mathematical Olympiad in China,Mathematical Olympiad Challenges, Mathematical Olympiad Treasures, International
Page 3: HU NSEN 3 STR O N G...Problems and Solutions . GñkcUlrYmRtYtBinitübec© ... Mathematical Olympiad in China,Mathematical Olympiad Challenges, Mathematical Olympiad Treasures, International

GñkcUlrYmRtYtBinitübec©keTs

elak lwm qun elak Esn Bisidæ elakRs I Tuy rINa elak Titü emg elak RBwm sunitü elak pl b‘unqay

GñkRtYtBinitüGkçraviruTæ elak lwm miKþsir

karIkMuBüÚT&r kBaØa lI KuNÑaka

GñkniBnæ nig eroberog elak lwm plþún nig elak Esn Bisidæ

Page 4: HU NSEN 3 STR O N G...Problems and Solutions . GñkcUlrYmRtYtBinitübec© ... Mathematical Olympiad in China,Mathematical Olympiad Challenges, Mathematical Olympiad Treasures, International

GarmÖkfa

esovePA KNitviTüaCMuvijBiPBelak EdlGñksikßakMBugkan´ enAkñúgéden£ xMJúáTáneroberogeLIgkñúgeKalbMngTukCaÉksar sRmab´ CaCMnYydl´GñksikßaykeTAsikßaRsavRCavedayxøÜnÉg nig müageTot kñúgeKalbMngcUlrYmelIksÞÜyvis&yKNitviTüaenARbeTskm<úCaeyIg [kan´EtrIkceRmInEfmeTotedIm,Ibeg;InFnFanmnusß[mankanEteRcIn edIm,ICYyGPivDÄn_RbeTsCatirbseyIg . enAkñúgesoven£eyIgxMJúánxitxMRsavRCaveRCIserIsyklMhat´ya¨g sRmaMgbMputBIÉksarbreTsyageRcInbMputdUcCa 103 Trigonometry Problems ,

Five Hundred Mathematical Challenges,Complex Numbers From A to … Z , Mathematical Olympiad in China,Mathematical Olympiad Challenges, Mathematical Olympiad Treasures, International Mathematical Olympiads 1959-1977,

The IMO Compendium ( 1959-2004 ) …nigÉksarbreTsepßg@eTottamy¼ Internet ykmkeFIVdMeNa£Rsayya¨gek,a£k,ayEdlGac[elakGñk

gayyl´qab´cgcaMGMBIsil,£énkareda£RsayTaMgGs´en£ . b¨uEnþeTa£Ca ya¨gNak¾eday kgV£xat nig kMhusqþgedayGectnaRákdCamanTaMg bec©keTs nig GkçraviruTæ . GaRs&yehtuen£ eyIgxMJúCaGñkeroberogrgcaMedayrIkrayCanic©nUv mtiri£Kn´EbbsSabnaBIsMNakGñksikßakñúgRKbmCÄdæanedIm,ICYyEklMG esovePAen£[ánkan´EtsuRkiRtPaBEfmeTot .

Page 5: HU NSEN 3 STR O N G...Problems and Solutions . GñkcUlrYmRtYtBinitübec© ... Mathematical Olympiad in China,Mathematical Olympiad Challenges, Mathematical Olympiad Treasures, International

CaTIbBa©b´en£eyIgxMJúGñkeroberogsUmeKarBCUnBrdlGñksikßaTaMgGs´ [mansuxPaBmaMmYn nig TTYlC&yCMn£RKb´Parkic© .

átdMbgéf¶TI07 mkra 2009 GñkniBnæ lwm plþún Tel : 017 768 246

Page 6: HU NSEN 3 STR O N G...Problems and Solutions . GñkcUlrYmRtYtBinitübec© ... Mathematical Olympiad in China,Mathematical Olympiad Challenges, Mathematical Olympiad Treasures, International

lwm plþún nig Esn Bisidæ

H U N S E N× 3

S T R O N G

Problems and Solutions

Page 7: HU NSEN 3 STR O N G...Problems and Solutions . GñkcUlrYmRtYtBinitübec© ... Mathematical Olympiad in China,Mathematical Olympiad Challenges, Mathematical Olympiad Treasures, International

rkßasiT§iRKb´yag

Page 8: HU NSEN 3 STR O N G...Problems and Solutions . GñkcUlrYmRtYtBinitübec© ... Mathematical Olympiad in China,Mathematical Olympiad Challenges, Mathematical Olympiad Treasures, International

KNitviTüaCMuvijBiPBelak

EpñkRbFanlMhat

- 1 - PaK1

Page 9: HU NSEN 3 STR O N G...Problems and Solutions . GñkcUlrYmRtYtBinitübec© ... Mathematical Olympiad in China,Mathematical Olympiad Challenges, Mathematical Olympiad Treasures, International

KNitviTüaCMuvijBiPBelak

- 2 - PaK1

EpñkRbFanlMhat´ 1-BIcMnYnBitviC¢man nig x y

epÞógpÞat´TMnak´TMng 11y3x4 =+ cUrkMntrtémøGtibrmaénGnuKmn_ ½

6 )3)(7y)(x()y,x(f y2x +++= 2 enøa¼-cUrkMntRKb´témø kñúgc x [

2;0] π edaydwgfa ½

2xcos13

xsin13 ++

− 4=

mYy3-enAkñúgRtIekaN Edl cAB,bAC,aBCABC === cUrbgHajfa

cba

2Asin

+≤ rYcsresrTMnak´TMngBIreTotEdl

tag[ce RsedogKña . 4-eKyk I nøa¼ ]

4;

4[ ππ− . cUrkMnt´GnuKmn_

os(s

f

kMntelI [ − ebIeKdwgfa cxsin)x2in]1,1 xf += 2 cMeBa¼RKb´ kñúgcenøa¼ .

jfa t

rYcsRmYl f )x(tan x I

5-cUrbgHa tanatana2tana3an atana2tana3=−− Edl

2ka π

≠ RKb´cMnYnKtrWTLaTIhV . k

Page 10: HU NSEN 3 STR O N G...Problems and Solutions . GñkcUlrYmRtYtBinitübec© ... Mathematical Olympiad in China,Mathematical Olympiad Challenges, Mathematical Olympiad Treasures, International

KNitviTüaCMuvijBiPBelak

- 3 - PaK1

6-cUrbgHajfa ( )2

ab21xcos

b1xsin

a1 +≥⎟⎠⎞

⎜⎝⎛ +⎟

⎠⎞

⎜⎝⎛ +

cMeBa¼RKb´ 2

xb,0a 0,0 π<>> <

-e ue mYyEdl 7 K[ct kaNEkg ABCD AB3BC = . aBIrcMnucénRCug Edl QB = P nig Q C PP]BC[ QC= .

cUrbgHajfa DPDBCDQC C∠+∠=∠ . CacMnYnkMupøicBIr . 8-eK[ 1z nig 2z

cUrRsayfa ( )22

21

221

22 2|zz||zz| 1 |z||z| +=−++

9-eK[ 2 Ca nig cMnYnkMupøicBIrEdl 1z z 1|z||z| 21 == . nig z.z 21 −≠ 1

cUrRsayfa 21

21

zz1+ CacMnYnBzz + itmYy .

10-KNnalImItxageRkam ½

4x

4x1212...121212lim

4x −

−++++++ man n ¦skaer .

Page 11: HU NSEN 3 STR O N G...Problems and Solutions . GñkcUlrYmRtYtBinitübec© ... Mathematical Olympiad in China,Mathematical Olympiad Challenges, Mathematical Olympiad Treasures, International

KNitviTüaCMuvijBiPBelak

- 4 - PaK1

11-eK[sIVúténcMnYnBit kMnt´eday ½

=

)u( n

0 nig 1u1u4 n +u6u4

uu 2n

3n

4n

1n ++=+ RKb´

INn∈

cUrbgHajfa4

n1n u1

u ⎟⎟⎠

⎜⎜⎝

+=++

111⎞⎛ RKb´ INn∈

CaGnuKmn_én . KNna

1)(1n2 +

+−

. nmMu kat g´cMnuc .

rYcKNna nu n

12- x1).....(xxxx1(P

n2422n −++−= )x

13-eK[RtIekaN ABC mYymanRCugRbEvg c,ba ,

knø¼bnÞatBu¼é ] RtC AB[ D

cUrRsayfa 2Ccos

baab2CD+

= . KNnaplbUk 14-

1Sn !n.n.....!3.3!2.2!1. ++++ =

Edl n(n!n 1.2.3)....2n)(1 −− . KNnatémø

=

15- )29tan3)...(3tan3)(2tan3)(1tan3(A oooo ++++=

Page 12: HU NSEN 3 STR O N G...Problems and Solutions . GñkcUlrYmRtYtBinitübec© ... Mathematical Olympiad in China,Mathematical Olympiad Challenges, Mathematical Olympiad Treasures, International

KNitviTüaCMuvijBiPBelak

- 5 - PaK1

16- itEdleK[ CacMnYnB epÞógpÞat´lkçx&NÐ ½ z,y,x

ycosxcos 0zcos =++ cos nig x3cos 0z3cosy3 =++ cx2s cUrbgHajfa 2cosy2osco 0z ≤ .

k ug .

cUrRsayfa ½

17-eK[RtIekaN mYy . ABC

bnÞat´Bu¼mMu A at´RCC,B, ]AB[,]AC[,]BC[

erogKñaRtg´ ,'A 'C,'B

0'2

'AA2 =

CC

)BAsin(

'BB

)2

ACsin()CBsin( −

+

+

18-eK[BhuFa n)asasinx()x(P co+= n∈Edl

nwg *IN

cUrrksMNlénviFIEckrvag )x(P 1x2 + .

&n19-eda¼RsayRbB æsmIkar ⎩⎨

− yyx4 ⎧

−=

+=+

baxbayx

4

3

CacMnYnBitEdl Edl a nig b 0ba ≠+ . 0-cUrbgHajfa 2 9

)1nn)(1nn(19999

1n44 =⎟

⎞⎜⎝

⎛++++

∑=

Page 13: HU NSEN 3 STR O N G...Problems and Solutions . GñkcUlrYmRtYtBinitübec© ... Mathematical Olympiad in China,Mathematical Olympiad Challenges, Mathematical Olympiad Treasures, International

KNitviTüaCMuvijBiPBelak

- 6 - PaK1

21- IeK[GnuKmn_ kMnt´RKb´ f }0;1{Rx −−∈ eday ½ 1x)

x1(f)x(f)1x2(x + +=+ .

cUrKNna )2009(f....)3(f)2(f)1(fS ++++= 22- éncMnYnKtcUrkMnt´RKb´KU ´viC¢manebIeKdwgfa ½ )n;m(

)m(13nm 22 n+=+ . 23-eK[RtIekaN C m g´ A .AB YyEkgRt

k nig eKy F]AC[E∈ ]AB[∈ Edl ABCAEF ∠=∠

nig erogKñaCaeCIgcMeNalEkgén nig .

ayfa

nig ACBAF ∠=∠ . E

eKtag E 'F ' E F

elIRCug BC[ ]

cUrRs BC'FFEFE'E ≤++ ?

24-eK[ CabIcMnYnBitviC¢man . cUrRsaybBa¢ak´fa ½ c,b,a

2)bb)cb(4a 33 33 a(4c

ba)ac(4

accb3 33 33

≤++

++

+++

+++

+

it =

25-eK[sIVútcMnYnB−⎩

⎨⎧

≥∀+

==

−+ 1n,2a2a1aa

1n1n

10 an

Page 14: HU NSEN 3 STR O N G...Problems and Solutions . GñkcUlrYmRtYtBinitübec© ... Mathematical Olympiad in China,Mathematical Olympiad Challenges, Mathematical Olympiad Treasures, International

KNitviTüaCMuvijBiPBelak

- 7 - PaK1

cUrRsayfa 1nn aaa +1n 2 +×= .

26-kñúgRtIekaN mYycUrRsaybBa¢akfa ½ ABC

r2Csin

2Bsin

2Asin

R4111≥++

ig Edl r n R C IekaN . CabIcMnYnBitviC¢manEdl akaMrgVg´carwkkñúg nig carwkeRkARt

27-eK[ b,a c, 1cbaabc4 +++= cUrbgHajfa ½

)cabcab(2c

bab

ca

cb≥

+++

+ a 222222

+++ 28-eK kMnt´eday ½

101n

[sIVút }a{ n

1a0 = nig 4na....a.aa +=+ a¼RKb . a

cMeB ´ n ≥ 1

cUrbgHajf 2a 1n =+ cMeBa¼RKb´ n ≥an − . CabIcMnYnBitviC¢manEdl cab

1

29-eK[ ,a bc,b 1ca =++ . cUrbgHajfa 16)

a1c()

c1b()

b1a( 222 ≥+++++

30-eK[ 432 222,

2cos222,

2cos22

2cos2 π

=++π

=+π

= IcUrrkrUbmnþTUeTA nig RsaybBa¢ak´

mnþena¼pg . BI«TahrN_xagelrUb

Page 15: HU NSEN 3 STR O N G...Problems and Solutions . GñkcUlrYmRtYtBinitübec© ... Mathematical Olympiad in China,Mathematical Olympiad Challenges, Mathematical Olympiad Treasures, International

KNitviTüaCMuvijBiPBelak

- 8 - PaK1

31-edayeFIVvicart

dac´nwg Canic©cMeBa¼RKb´ μCati n .

n,7 ∈

amkMeNIncUrRsaybBa¢akfacMnYn ½1n1n6

n 92A ++ += Eck 11

cMnYnKtFm32-eK[ N61009743831E nnnn

n I−+−=

acnwg Canic©RKb´ .

cUrbgHajfa nE Eckd 187 INn∈

33-cUrRsaybBa¢akfa 1n21

1 <+n1.....

31

21

++

+++ viC¢man .

cUrRsaybBa¢akfa ½ 34-eK[ n cMnYnBit n321 a.........;;a;a;a

nn321n321 na..........aaa ≥++++ a......a.a.a

35-eda¼RsayRbB&næsmIkar ½

⎩⎨

tanxtan 33

=+

=++

34ytanytanx28ytanytanxtan6xtan 4224

Edl 2

x0 π< . y <<

´RtIekaNmYynigtag énRtIekaN .

36-eK[ c,b,a CaRCugrbs p

Caknø¼brimaRt

Page 16: HU NSEN 3 STR O N G...Problems and Solutions . GñkcUlrYmRtYtBinitübec© ... Mathematical Olympiad in China,Mathematical Olympiad Challenges, Mathematical Olympiad Treasures, International

KNitviTüaCMuvijBiPBelak

- 9 - PaK1

cUrbgHajfa ⎟⎠⎞

⎜⎝⎛ ++≥

−+

−+

− bp1

ap1

c1

b1

a12

cp1

-k¿cUrRsaybBa¢akfa ½ 37

3n41n4

1n41n4

1n43n4

<+−

<+−

+− cMeBa¼RKb´

x¿TajbgHajfa ½ *INn∈

3n43

)1n4(.......139)1n4(.......117

1n21

+×××−×××

+ 53

+<

××

<

CabIcMnYnBitviC¢man . 38-eK[ ;b;a c

cUrbgHajfa 23

bac

acb

cba

≥+

++

++

39-k¿KNna n)

....2.1

1++ Edl 2n >

1n(1

4.31

3.21

−++

x¿edayeRbIvismPaB GMAM − én n( )1− cMnYnxageRkam ½

n)11;

3.21;

2.11

− cUrbgHajfa nn

n(;

4.31 )!n( 2< .

CabIcMnYnBitxusBIsUnü .

fa

40-eK[ b;a c;

cUrbgHajac

cb

ba

ac

cb 222

++≥⎟⎠⎞

⎜⎝⎛+⎟

⎞⎜⎝⎛+⎟

⎞ ba

⎠⎠⎜⎝⎛

CabIcMnYnBitviC¢manminsUcUrbgHajfa

41-eK[ y;x nü . z;

222222 zyx9

z1

y1

x1

++≥++

Page 17: HU NSEN 3 STR O N G...Problems and Solutions . GñkcUlrYmRtYtBinitübec© ... Mathematical Olympiad in China,Mathematical Olympiad Challenges, Mathematical Olympiad Treasures, International

KNitviTüaCMuvijBiPBelak

- 10 - PaK1

42-eKman xlog11

aay nig −= ylog11

aaz −=

cUrRsayfa zlog11

aax −= .

43-eKBinitüsIVúténcMnYnBit kMnt´eday )u( n⎪⎩

⎪⎨

−−

=

=

+ 1u24u5

u

4u

n

n1n

1

cUrKNna CaGnuKmn_én .

44-eKBinitüsIVúténcMnYnBit kMnt´eday

nu n

)u( n⎪⎩

⎪⎨

−−

=

=

+ 1u4u3

u

3u

n

n1n

1

45-eKBinitüsIVúténcMnYnBit n kMnt´eday ½ cUrKNna nu CaGnuKmn_én . n

u( )

⎩⎨

∈=+ *INn,uu 1n

⎧ = 4u

n

1

_én 46-eK[ Bit kMnt´eday ½ cUrKNna nu CaGnuKmn n .

sIVúténcMnYn )u( n

5u0 = nig 5nu21u nn ++ ( INn1 =+ ∈ )

rKNna énsIVút CaGnuKmn_én . cU nu )u( n n

Page 18: HU NSEN 3 STR O N G...Problems and Solutions . GñkcUlrYmRtYtBinitübec© ... Mathematical Olympiad in China,Mathematical Olympiad Challenges, Mathematical Olympiad Treasures, International

KNitviTüaCMuvijBiPBelak

- 11 - PaK1

47-edayeRbIGnumanrYmKNitviTüacUrKNnalImItxageRkam ½

2x

2x22.....222limL

2xn −−++++

.

8-eK[sIVúténcMnYnBit k t´elI eday ½

++=

( man n ra¨DIkal´ ) 4 Mn)U( n IN

⎪⎩⎨

∈+=+ INn,UU n1n

0 ⎪⎧ =

2

2U

GnuKmn_én . ..U

k¿cUrKNna nU Ca n

x¿KNnaplKuN n21n U..UUP 0 ××××= . 49-eK[s MnYnBit kMnt´elI eday ½IVúténc )U( n IN

22U0 = ig n INn,

2U11

U 1n =+

2n ∈∀

−−

0-eK[sIVúténcMnYnBit kMnt´eday½

=

2a,INn,

au0

KNna nU CaGnuKmn_én . n

5 )u( n

⎩ −=+ 2uu 2n1n

⎨⎧

>∈∀

Page 19: HU NSEN 3 STR O N G...Problems and Solutions . GñkcUlrYmRtYtBinitübec© ... Mathematical Olympiad in China,Mathematical Olympiad Challenges, Mathematical Olympiad Treasures, International

KNitviTüaCMuvijBiPBelak

- 12 - PaK1

cUrRsaybBa¢ak´fa nn

4aau ⎟⎞

⎜⎛ −

+⎜⎛ +

=2

22

2

n 2a

24a

⎟⎠

⎜⎝

−⎟⎟⎠

⎞⎜⎝

51-k¿cUrkMntelxénGBaØat éncMnYn d,c,b,a abcd ebIeKdwgfa ½

dcbabcd = a9×

cMnYn x¿cMeBa¼témø EdlánrkeXIjxagelIcUrbBa¢ak´fa d,c,b,a

abcd nig dcba suTæEtCakaer®ákd . 52-snμtfa HUNSEN Cac YnmYymanelxRáMmYyxÞg´Edl Mn

Caelx . KuNcMnYne eKTTYl CacMnYnmanelx

Þg´KW

N,E,S,N,U,H

eBlEdleK n¼nwg Tæpl3

RáMmYyx STRONG Edl G,N,O,R,T,S Caelx cUrrkcMnYn HUNSEN nig STRONG cMeBa¼ 3S,6N == nigcMeBa¼ 8N 9S, == . (GkßrxusKñatagedayelxxusKña ). 53-cUrbMeBjRbmaNviFIbUkxageRkam ½

Page 20: HU NSEN 3 STR O N G...Problems and Solutions . GñkcUlrYmRtYtBinitübec© ... Mathematical Olympiad in China,Mathematical Olympiad Challenges, Mathematical Olympiad Treasures, International

KNitviTüaCMuvijBiPBelak

- 13 - PaK1

YENOM EROM

E

(GkßrxusKñatagedayelxxusKYn ´Ed

DNS+

ña ) 54-eK[ACacMn mYymanelxRáMxÞg lelxxÞg´vaerobtam lMdab´ x1x;x ;1x;2x; ++ CacMnYnmYymanelx

lelxxÞg+ ehIy B

RáMmYyxÞg´Ed ´vaerobtamlMdab´ x;1x;1x;2x;1x;x −+ .

cUrbgHajfae CakaerRákdena¼ B k¾Cakaer®ákdEdr .++

bI A 55-eK[cMnYn

)elxn2(

111.....111A = nig B) elxn(

444.....444=

++ CakaerRákd . MnYnmYymanelxbYnxÞg´EdlelxxÞg´vaerobtamlMdab´

.

7-eKman

cUrbgHajfa A 1B

56-c b;b;a;a

rkcMnYnena¼ebIeKdwgfavaCakaerRákd . 5 111088893333,110889333,108933 222 ===

Page 21: HU NSEN 3 STR O N G...Problems and Solutions . GñkcUlrYmRtYtBinitübec© ... Mathematical Olympiad in China,Mathematical Olympiad Challenges, Mathematical Olympiad Treasures, International

KNitviTüaCMuvijBiPBelak

- 14 - PaK1

333332 1111088889= . ahrN_xagelIcUrrkrUbmnþTUeTAnigRsaybBa¢ak´rUbmnþen¼pg. BI«T

58-eKman 999800019999,998001999,980199 222 === . 9999800001999992 =

BI«TahrN_xagelIcUrrkrUbmnþTUeTAnigRsaybBa¢ak´rUbmnþen¼pg. 59-eKman 444355566666,443556666,435666 222 ===

4556,115 22222 =

4444355556666662 = . BI«TahrN_xagelIcUrrkrUbmnþTUeTAnigRsaybBa¢ak´rUbmnþen¼pg 60- eKman

111111445556,111162 −=−=

«TahrN_xagelIcUrrkrUbmnþTUeTA nigRsaybBa¢ak´rUbmnþen¼pg

1111111144455556 22 =− . BI

Page 22: HU NSEN 3 STR O N G...Problems and Solutions . GñkcUlrYmRtYtBinitübec© ... Mathematical Olympiad in China,Mathematical Olympiad Challenges, Mathematical Olympiad Treasures, International

KNitviTüaCMuvijBiPBelak

- 15 - PaK1

61-eK[ 011nn aa..........aaA = − nig 0a2B ×− 11nn a.........aa= −

ac´nwg . Rsayfa A Eckdac´nwg 7 lu¼RtaEt B Eckd 7

62-eK[cMnYn 012nn aaa......aaA − 1=

,

nYn ac´nwg kalNa a)a...

Edl n210 a....,aa,a Caelx . .,

cUrRsayfacM A Eckd 6

0n21 aa(4y ++ Eckdacnwg nig Im,I[cMnYn

++= . 6

63-cUrkMnt´elx b eda abba CaKUbéncMnYnKt 64-eKtag r nig R erogKñaCakaMrgVg´carikkñúg nigcarikeRkA énRtIekaN ABCmYy . k¿cUrRsayfa

Rr1CcosBcosAcos +=++

. x¿cUrRsayfa R ≥ r2

65-k¿cUrRsayfa x2cotxcotx2sin

1−=

¿cUrKNna xn2

n

2a

sin

1....

2a

sin

1

2a

sin

1asin

1S ++++=

Page 23: HU NSEN 3 STR O N G...Problems and Solutions . GñkcUlrYmRtYtBinitübec© ... Mathematical Olympiad in China,Mathematical Olympiad Challenges, Mathematical Olympiad Treasures, International

KNitviTüaCMuvijBiPBelak

- 16 - PaK1

66-k¿cUrRsayfa xcot2x

cot11+

xcos=

¿KNn uN x aplK )

2a

cos

11).a

cos

11)(a

cos

11)(

ac1(Pn +++= ....(

22os1

n2

+

Bit n

67-eK[sIVúténcMnYn )U( kMnt´elI IN eday ½

2U0 = nig 2

INn,2

U11Un+

2n

1 ∈∀−−

= n_én .

cUrKNna KNna nU CaGnuKm n

68-9π7

cos9π4

cos9π

cosS 333 +−= 69-eK[RtIekaN mYymanmMukñúgCamMuRsYc . ABC

k¿cUrRsayfa Ctan.Btan.AtanBtanAtan Ctan =++ x¿TajbBa¢akfa 33CtanBtanAtan ≥++ .

½

70-KNnaplbUk

)9n(999......999....999999Sn

elx++++=

IeKdwgfacMnYn ½ 71-cUrkMnt´RKb´KUtémøKt´viC¢man ) ebb,a(

1bab2

a32 +

CacMnYnKt´viC¢manE2

−dr .

Page 24: HU NSEN 3 STR O N G...Problems and Solutions . GñkcUlrYmRtYtBinitübec© ... Mathematical Olympiad in China,Mathematical Olympiad Challenges, Mathematical Olympiad Treasures, International

KNitviTüaCMuvijBiPBelak

- 17 - PaK1

72-cUrbgHajfa ½ 1

ab8cca8bbc8a 222≥

cba+

++

++

cMeBa¼RK b´cMnYnBitviC¢man .

YnBitEdlc,b,a

73-eK[ )a( n CasIVúténcMn 21a1 =

nigcMeBa¼RKb´cMnYnKt´v ¢mann eyIgmiC an 1aa

a2

2n

+−= a

nn1n+

´cMnYnKt n eyIgman ½n

cUrRsaybBa¢ak´facMeBa¼RKb ´viC¢man aaa 321 1a......... <++++ .

øKt´ ebIeKdwgfacMeBa¼RKb´ MnYnKt´viC¢man eKman

74-cUrkMnt´RKb´KUtém 3n,m ≥

c a1aa1aa

2n

m

−+−+ CacMnYnKt´ .

YnKt´viC¢man Edl =+75-eK[bIcMn ac,b,a 10cb+ . n cUrrktémøFMbMputé cbaP ××= .

76-eKmansmPaB ba

1b

xcosa

xsin 44

+=+

Edl ,0a 0ba,0b ≠+≠ ≠

cUrbgHajfa 33

8

3

8

)ba(1+b

xcosa

xsin=+ .

Page 25: HU NSEN 3 STR O N G...Problems and Solutions . GñkcUlrYmRtYtBinitübec© ... Mathematical Olympiad in China,Mathematical Olympiad Challenges, Mathematical Olympiad Treasures, International

KNitviTüaCMuvijBiPBelak

- 18 - PaK1

77-eK[RtIekaN ABC mYYy . bgHajfaebI

3Ctan,

3Btan tan,

3A

ax2Ca¦ssmIkar cbxx:)E( 3 0=+++ án ena¼eK cb3a =+3 + .

atémø®ákdén 78-k¿ cUrKNn10

sin π nig 10

cos π a x¿ cUrRsayf

10sin)yx(4)yx(x 22222 π

+≤−+ RKb´cMnYn IRy,x ∈ .

79- eK[sIVúténcMnYnBit kMnt´)U( n eday4

nsin.2Un

n π=

ðajfa Edl *In∈ . N

k-cUrbg4

nsin4

ncos4

)1n(cos.2 π−

π=

π+ x-Taj[ánfa

4)1n(c1 os)2(ncos)2(U nn

n 4π+

−π

= bUk ½

U.........U

+

K-KNnapln321n UUS ++++= _éCaGnuKmn n .

n32 ≥ . n

80-eK[ n cMnYnBit a1 0a.......;;a;a;

eKtag *INn,n

a.....aaaS 1

nn32 ∈∀

++++ =

Page 26: HU NSEN 3 STR O N G...Problems and Solutions . GñkcUlrYmRtYtBinitübec© ... Mathematical Olympiad in China,Mathematical Olympiad Challenges, Mathematical Olympiad Treasures, International

KNitviTüaCMuvijBiPBelak

- 19 - PaK1

cUrRsayfaebI 1n1nn3211n aa.......a.a.aS +

++ ≥ ena¼eKán n

n31 a.....a≥ . 2n aaS

gHajfa 81-cUrb101

10099......

65.

43.

21

< 151<

82-eK[ cMnYnB 1n2 b.;b;b;a.....;;a .

2n

22

21

2n

222nn2211 ++++++≤+++

83-eK[ nig CasIVútcMnYnBitkMnt´elI eday nigTMnak´TMngkMenIn ½

nig RKb´ cUrKNna nig CaGnuKmn_én .

n2 it 21 ...;;a n

cUrRsayfa ½ )b.....bb)(a....aa()ba....baba( 21

)x( n )y( n IN

1y,5x 00 ==2

nn3

n1n yx3xx +=+3

nn2

n1n yyx3y +=+ INn∈

nx ny n

Page 27: HU NSEN 3 STR O N G...Problems and Solutions . GñkcUlrYmRtYtBinitübec© ... Mathematical Olympiad in China,Mathematical Olympiad Challenges, Mathematical Olympiad Treasures, International

KNitviTüaCMuvijBiPBelak

EpñkdMeNa¼Rsay

- 20 - PaK1

Page 28: HU NSEN 3 STR O N G...Problems and Solutions . GñkcUlrYmRtYtBinitübec© ... Mathematical Olympiad in China,Mathematical Olympiad Challenges, Mathematical Olympiad Treasures, International

KNitviTüaCMuvijBiPBelak

lMhat´TI1 BIcMnYnBitviC¢man nig epÞógpÞat´TMnak´TMngx y 11y3x4 =+ cUrkMntrtémøGtibrmaénGnuKmn_ ½

)y2x3)(7y)(6x()y,x(f +++= dMeNa¼Rsay kMnt´rtémøGtibrmaénGnuKmn_ ½

)y2x3)(7y)(6x()y,x(f +++= tamvismPaB eyIgán ½ GMAM −

3

3

)y,x(f3

13y3x4

)y2x3)(7y)(6x(3

)y2x3()7y()6x(

≥++

+++≥+++++

eKTaj 3

313y3x4)y,x(f ⎟

⎠⎞

⎜⎝⎛ ++

≤ eday 11y3x4 =+

eKán 5123

1311)y,x(f3

=⎟⎠⎞

⎜⎝⎛ +

≤ . dUcen¼témøGtibrmaénGnuKmn_ )y,x(f es μInwg . 512

- 21 - PaK1

Page 29: HU NSEN 3 STR O N G...Problems and Solutions . GñkcUlrYmRtYtBinitübec© ... Mathematical Olympiad in China,Mathematical Olympiad Challenges, Mathematical Olympiad Treasures, International

KNitviTüaCMuvijBiPBelak

- 22 - PaK1

lMhat´TI2 cUrkMntRKb´témø kñúgcenøa¼ x [

2;0] π edaydwgfa ½

24xcos13

xsin13

=+

+−

dMeNa¼Rsay kMnt´RKb´témø kñúgcenøa¼ x [

2;0] π

)1(24xcos13

xsin13

=+

+−

eKBinitü ⎟⎠⎞

⎜⎝⎛ π

−π

43sin

12sin

4

2621.

22

22.

23

3cos

4sin

4cos

3sin

−=−=

ππ−

ππ=

ehIy ⎟⎠⎞

⎜⎝⎛ π

−π

43cos

12cos

4

2622.

23

22.

21

2sin

3sin

4cos

3cos

+=+=

ππ+

ππ=

Page 30: HU NSEN 3 STR O N G...Problems and Solutions . GñkcUlrYmRtYtBinitübec© ... Mathematical Olympiad in China,Mathematical Olympiad Challenges, Mathematical Olympiad Treasures, International

KNitviTüaCMuvijBiPBelak

smIkar GacsresreTACa ½ )1(

x2sinx12

sin

xcosxsin212

cosxsinxcos12

sin

2xcos

12cos

xsin12

sin

2xcos

426

xsin4

26

2xcos22

13

xsin22

13

=⎟⎠⎞

⎜⎝⎛ +π

=

π

+

π

=

+

+

=

+

+

eday 2

x0 π<< ena¼eKTaján

⎢⎢⎢⎢

−π=+π

=+π

x2x12

x2x12

eKTaj 12

x π= ¦

3611x π

= ( epÞógpÞat´nwglkçx&NÐEdl[ ) dUcen¼ }

3611;

12{x ππ

∈ .

- 23 - PaK1

Page 31: HU NSEN 3 STR O N G...Problems and Solutions . GñkcUlrYmRtYtBinitübec© ... Mathematical Olympiad in China,Mathematical Olympiad Challenges, Mathematical Olympiad Treasures, International

KNitviTüaCMuvijBiPBelak

- 24 - PaK1

lMhat´TI3 enAkñúgRtIekaN mYyEdl ABC cAB,bAC,aBC === cUrbgHajfa

cba

2Asin

+≤ rYcsresrTMnak´TMngBIreTotEdl

RsedogKña .

dMeNa¼Rsay bgHajfa

cba

2Asin

+≤

tamRTwsþIbTsIunUs R2Csin

cBsin

bAsin

a===

Edl R CakaMrgVg´carwkeRkARtIekaN . ABC

eKTaj CsinR2c,BsinR2b,AsinR2a === eKman

CsinR2BsinR2AsinR2

cba

+=

+

2CBcos)

2A

2sin(

2Acos

2Asin

cba

2CBcos

2CBsin2

2Acos

2Asin2

CsinBsinAsin

cba

−−

π=

+

−+=

+=

+

Page 32: HU NSEN 3 STR O N G...Problems and Solutions . GñkcUlrYmRtYtBinitübec© ... Mathematical Olympiad in China,Mathematical Olympiad Challenges, Mathematical Olympiad Treasures, International

KNitviTüaCMuvijBiPBelak

( )1

2CBcos

2Asin

2CBcos

2Acos

2Acos

2Asin

cba

−=

−=

+

eday π<−≤ |CB|0 ena¼ 12

CBcos0 ≤−

<

tam eKTaj )1(2Asin

2CBcos

2Asin

cba

≥−

=+

.

dUcen¼ cb

a2Asin

+≤ .

TMnak´TMngBIreTotEdlRsedogKñaen¼KW ½

acb

2Bsin

+≤ nig

bac

2Csin

+≤ .

- 25 - PaK1

Page 33: HU NSEN 3 STR O N G...Problems and Solutions . GñkcUlrYmRtYtBinitübec© ... Mathematical Olympiad in China,Mathematical Olympiad Challenges, Mathematical Olympiad Treasures, International

KNitviTüaCMuvijBiPBelak

- 26 - PaK1

lMhat´TI4 eKyk tag[cenøa¼ I ]

4;

4[ ππ−

. cUrkMnt´GnuKmn_ f kMnt´elI [ ebIeKdwgfa ]1,1 xcosxsin)x2(sinf += rYcsRmYl f cMeBa¼RKb´ x kñúgcenøa¼ I . )x(tan2

dMeNa¼Rsay kMnt´GnuKmn_ f eKman xcosxsin)x2(sinf += elICakaer f 22 )xcosx(sin)x2(sin +=

x2sin1)x2(sinfxcosxcosxsin2xsin)x2(sinf

2

222

+=

++=

eday )x4

sin(2xcosxsin +π

=+ cMeBa¼RKb´ Ix∈

eKán 0)x2(sinf > . ehtuen¼ x2sin1)x2(sinf += tag x2sinz = dUcen¼ z1)z(f += cMeBa¼ 1z1 <<− . müa¨geTotRKb´ ]

4,

4[x ππ−∈ eKman 1xtan1 ≤≤−

Page 34: HU NSEN 3 STR O N G...Problems and Solutions . GñkcUlrYmRtYtBinitübec© ... Mathematical Olympiad in China,Mathematical Olympiad Challenges, Mathematical Olympiad Treasures, International

KNitviTüaCMuvijBiPBelak

naM[ . 1xtan0 2 ≤≤

dUcen¼ xcos

1xcos

1xtan1)x(tanf 222 ==+= .

lMhat´TI5 cUrbgHajfa atana2tana3tanatana2tana3tan =−− Edl

2ka π

≠ RKb´cMnYnKtrWTLaTIhV k .

dMeNa¼Rsay bgHajfa atana2tana3tanatana2tana3tan =−− eKman )aa2tan(a3tan +=

atana2tanatana2tana3tana3tan

atana2tan)atana2tan1(a3tanatana2tan1atana2tana3tan

+=−+=−

−+

=

dYcen¼ atana2tana3tanatana2tana3tan =−− .

- 27 - PaK1

Page 35: HU NSEN 3 STR O N G...Problems and Solutions . GñkcUlrYmRtYtBinitübec© ... Mathematical Olympiad in China,Mathematical Olympiad Challenges, Mathematical Olympiad Treasures, International

KNitviTüaCMuvijBiPBelak

- 28 - PaK1

lMhat´TI6 cUrbgHajfa ( )2

ab21xcos

b1xsin

a1 +≥⎟⎠⎞

⎜⎝⎛ +⎟

⎠⎞

⎜⎝⎛ +

cMeBa¼RKb´ 2

x0,0b,0a π<<>>

dMeNa¼Rsay bgHajfa ( )2

ab21xcos

b1xsin

a1 +≥⎟⎠⎞

⎜⎝⎛ +⎟

⎠⎞

⎜⎝⎛ +

eKman xcosxsin

abxcos

bxsin

a1)xcos

b1()xsin

a1( +++=++ tamvismPaB eKman ½ GMAM −

xcosxsinab2

xcosb

xsina

≥+

eKán xcosxsin

abxcosxsin

ab21)xcos

b1)(xsin

a1( ++≥++

22

2

)ab21()x2sin

ab21()xcos

b1()xsin

a1(

)xcosxsin

ab1()xcos

b1)(xsin

a1(

+≥+≥++

+≥++

BIeRBa¼RKb´ 2

x0 π<< eKman 1x2sin ≤ .

Page 36: HU NSEN 3 STR O N G...Problems and Solutions . GñkcUlrYmRtYtBinitübec© ... Mathematical Olympiad in China,Mathematical Olympiad Challenges, Mathematical Olympiad Treasures, International

KNitviTüaCMuvijBiPBelak

- 29 - PaK1

lMhat´TI7 eK[ctuekaNEkg mYyEdl BCABCD AB3= . P nig Q CaBIrcMnucénRCug Edl]BC[ QCPQBP == . cUrbgHajfa DPCDBCDQC ∠+∠=∠ .

dMeNa¼Rsay bgHajfa DPCDBCDQC ∠+∠=∠ tag γ=∠β=∠α=∠ DQC;DPC;DBC

eyIgman 1tan,21tan,

31tan =γ=β=α

eday γ==−

+=

βα−β+α

=β+α tan1

61131

21

tantan1tantan)tan(

dUcen¼ α ¦ γ=β+ DPCDBCDQC ∠+∠=∠ .

A

B C

D

P Q

Page 37: HU NSEN 3 STR O N G...Problems and Solutions . GñkcUlrYmRtYtBinitübec© ... Mathematical Olympiad in China,Mathematical Olympiad Challenges, Mathematical Olympiad Treasures, International

KNitviTüaCMuvijBiPBelak

- 30 - PaK1

lMhat´TI8 eK[ nig CacMnYnkMupøicBIr . 1z 2z

cUrRsayfa ( )22

21

221

221 |z||z|2|zz||zz| +=−++

dMeNa¼Rsay Rsayfa ( )2

22

12

212

21 |z||z|2|zz||zz| +=−++ eKman )zz)(zz(|zz| 2121

221 ++=+

( )1|z|zzzz|z||zz|

| zzzzzzzz|zz2

221212

12

21

222121112

21

+++=+

+++=+

ehIy )zz)(zz(|zz| 21212

21 −−=−

( )2|z|zzzz|z||zz|zzzzzzzz|zz|

222121

21

221

222121112

21

+−−=−

+−−=−

bUkTMnak´TMng ( nig )1 ( )2 eKán ½ ( )2

22

12

212

21 |z||z|2|zz||zz| +=−++ .

Page 38: HU NSEN 3 STR O N G...Problems and Solutions . GñkcUlrYmRtYtBinitübec© ... Mathematical Olympiad in China,Mathematical Olympiad Challenges, Mathematical Olympiad Treasures, International

KNitviTüaCMuvijBiPBelak

- 31 - PaK1

lMhat´TI9 eK[ nig CacMnYnkMupøicBIrEdl 1z 2z 1|z||z| 21 == nig z . 1z. 21 −≠

cUrRsayfa 21

21

zz1zz

++ CacMnYnBitmYy .

dMeNa¼Rsay Rsayfa

21

21

zz1zz

++ CacMnYnBitmYy

tag 21

21

zz1zzZ ena¼

++

=21

21

z.z1zzZ

++

=

eday 1|z|z.z 2111 == ena¼

11 z

1z = ehIydUcKña 2

2 z1z =

eKán Z1zz

zz

z1.

z11

z1

z1

Z12

12

21

21 =+

+=

+

+=

eday ZZ = ena¼ 21

21

zz1zzZ

++

= CacMnYnBit .

Page 39: HU NSEN 3 STR O N G...Problems and Solutions . GñkcUlrYmRtYtBinitübec© ... Mathematical Olympiad in China,Mathematical Olympiad Challenges, Mathematical Olympiad Treasures, International

KNitviTüaCMuvijBiPBelak

- 32 - PaK1

lMhat´TI10 KNnalImItxageRkam ½

4x4x1212...121212

lim4x −

−++++++→

man n ¦skaer .

dMeNa¼Rsay KNnalImIt

tag 4x

4x1212...121212limL

4xn −

−++++++=

( man ¦skaer ) n

eKán

4x4x1212...1212limL

4x1n −−+++++

=→−

( man ¦skaer ) 1n −

snμtfa 4x1212...121212)x(fn −++++++= nig 4x1212...121212)x(f n +++++++=

Page 40: HU NSEN 3 STR O N G...Problems and Solutions . GñkcUlrYmRtYtBinitübec© ... Mathematical Olympiad in China,Mathematical Olympiad Challenges, Mathematical Olympiad Treasures, International

KNitviTüaCMuvijBiPBelak

eKman ½

)x(f4x1212.....1212)x(f).x(f

16x1212...121212)x(f).x(f

1nnn

nn

−=−+++++=

−++++++=

eKTaj )x(f)x(f)x(f

n

1nn

−=

eKán )x(f

1lim.4x

)x(flim)x(f)4x(

)x(flimLn

4x

1n

4xn

1n

4xn →

→ −=

−=

)4(f

1.LLn

1nn −= eday 8)4(fn =

eKán 1nn L81L −= naM[ CasIVútFrNImaRtmanersug )L( n

81q = nigtY

4x4x12limL

4x1 −−+

=→

. tamrUbmnþtYTI n eKán 1n

1n qLL −×= eday

81

4x121lim

4x4x12limL

4x4x1 =++

=−

−+=

→→

nig 81q = ena¼ nn 8

1L = . dUcen¼

n4x 81

4x4x1212...121212

lim =−

−++++++→

.

- 33 - PaK1

Page 41: HU NSEN 3 STR O N G...Problems and Solutions . GñkcUlrYmRtYtBinitübec© ... Mathematical Olympiad in China,Mathematical Olympiad Challenges, Mathematical Olympiad Treasures, International

KNitviTüaCMuvijBiPBelak

- 34 - PaK1

lMhat´TI11 eK[sIVúténcMnYnBit kMnt´eday ½ )u( n

1u0 = nig 1u4u6u4

uu RKb´ n

2n

3n

4n

1n +++=+ INn∈

cUrbgHajfa 4

n1n u11

u11 RKb´ ⎟⎟

⎞⎜⎜⎝

⎛+=+

+

INn∈

rYcKNna u CaGnuKmn_én . n n

dMeNa¼Rsay

bgHajfa 4

n1n u11

u11 ⎟⎟

⎞⎜⎜⎝

⎛+=+

+

eyIgman 4n

n2

n3

n

1n u1u4u6u41

u11 +++

+=++

4

n4

n

4n

4n

n2

n3

n4

n

u11

u)1u(

u1u4u6u4u

⎟⎟⎠

⎞⎜⎜⎝

⎛+=

+=

++++=

dUcen¼ 4

n1n u11

u11 ⎟⎟

⎞⎜⎜⎝

⎛+=+

+

.

Page 42: HU NSEN 3 STR O N G...Problems and Solutions . GñkcUlrYmRtYtBinitübec© ... Mathematical Olympiad in China,Mathematical Olympiad Challenges, Mathematical Olympiad Treasures, International

KNitviTüaCMuvijBiPBelak

KNna CaGnuKmn_én nu n

eyIgman 4

n1n u11

u11 ⎟⎟

⎞⎜⎜⎝

⎛+=+

+

eKán ⎟⎟⎠

⎞⎜⎜⎝

⎛+=⎟⎟

⎞⎜⎜⎝

⎛+

+ n1n u11ln4

u11ln

tag ⎟⎟⎠

⎞⎜⎜⎝

⎛+=

nn u

11lnv naM[ ⎟⎟⎠

⎞⎜⎜⎝

⎛+=

++

1n1n u

11lnv

eKán . n1n v4v =+

TMnak´TMngen¼bBa¢ak´fa CasIVútFrNImaRtmanersug )v( n 4q = nigtY 2ln

u11lnv

00 =⎟⎟

⎞⎜⎜⎝

⎛+= ( eRBa¼ 1u0 = ) .

tamrUbmnþ 2ln4q.vv nn0n == .

eday ⎟⎟⎠

⎞⎜⎜⎝

⎛+=

nn u

11lnv

eKTaj 2ln4u11ln n

n

=⎟⎟⎠

⎞⎜⎜⎝

⎛+ naM[ n4

n

2u11 =+

dUcen¼ 12

1u n4n−

= .

- 35 - PaK1

Page 43: HU NSEN 3 STR O N G...Problems and Solutions . GñkcUlrYmRtYtBinitübec© ... Mathematical Olympiad in China,Mathematical Olympiad Challenges, Mathematical Olympiad Treasures, International

KNitviTüaCMuvijBiPBelak

lMhat´TI12 KNna

)xx1).....(xx1)(xx1(P1nn 22422

n

++−+−+−=

dMeNa¼Rsay KNna

)xx1).....(xx1)(xx1(P1nn 22422

n

++−+−+−=

eKman )aa1)(aa1(a)a1(aa1 2222242 +++−=−+=++ eKTaj 2

422

aa1aa1aa1

++++

=+− yk k2xa =

eKán 1kk

2k1k1kk

22

2222

xx1xx1xx1 +

+++

++

++=+−

2

22n

0k 22

22

n xx1xx1

xx1xx1P

2n1n

1kk

2k1k

++++

=⎟⎟⎠

⎞⎜⎜⎝

++

++=

++

+

++

∏=

dUcen¼ 2

22

n xx1xx1P

2n1n

++++

=++

.

- 36 - PaK1

Page 44: HU NSEN 3 STR O N G...Problems and Solutions . GñkcUlrYmRtYtBinitübec© ... Mathematical Olympiad in China,Mathematical Olympiad Challenges, Mathematical Olympiad Treasures, International

KNitviTüaCMuvijBiPBelak

- 37 - PaK1

lMhat´TI13 eK[RtIekaN mYymanRCugRbEvg . ABC c,b,a

knø¼bnÞat´Bu¼énmMu C kat´ [ Rtg´cMnuc D . ]AB

cUrRsayfa 2Ccos

baab2CD+

= .

dMeNa¼Rsay Rsayfa

2Ccos

baab2CD +

=

eyIgman S CDBACDABC SS += eday Csinab

21SABC =

2CsinCDb

21SACD = nig

2CsinCDa

21SCDB =

eKán 2CsinCD.a

21

2CsinCD.b

21sibCab

21

+= eKTaj

2Ccos

baab2

2Csin)ba(

CsinabCD+

=+

= .

dUcen¼ 2Ccos

baab2CD+

= .

C

A D B

Page 45: HU NSEN 3 STR O N G...Problems and Solutions . GñkcUlrYmRtYtBinitübec© ... Mathematical Olympiad in China,Mathematical Olympiad Challenges, Mathematical Olympiad Treasures, International

KNitviTüaCMuvijBiPBelak

- 38 - PaK1

lMhat´TI14 KNnaplbUk

!n.n.....!3.3!2.2!1.1Sn ++++= Edl 1.2.3)....2n)(1n(n!n −−= .

dMeNa¼Rsay KNnaplbUk

!n.n.....!3.3!2.2!1.1Sn ++++= eKman !k.]1)1k[(!k.k −+=

!k)!1k(!k.k!k!k).1k(!k.k

−+=−+=

eKán !n)!1n(...)!3!4()!2!3()!1!2(Sn −+++−+−+−= dUcen¼ 1)!1n(Sn −+= .

Page 46: HU NSEN 3 STR O N G...Problems and Solutions . GñkcUlrYmRtYtBinitübec© ... Mathematical Olympiad in China,Mathematical Olympiad Challenges, Mathematical Olympiad Treasures, International

KNitviTüaCMuvijBiPBelak

lMhat´TI15 KNnatémø

)29tan3)...(3tan3)(2tan3)(1tan3(A oooo ++++=

dMeNa¼Rsay KNnatémø A

)29tan3)...(3tan3)(2tan3)(1tan3(A oooo ++++= eKman oo

ooo0

1cos60cos61sin1tan60tan1tan3 =+=+

oo

ooo0

oo

oooo

oo

oooo

29cos60cos89sin29tan60tan29tan3

3cos60cos63sin3tan60tan3tan3

2cos60cos62sin2tan60tan2tan3

=+=+

−−−−−−−−−−−−−−−−−−−−−−−−

=+=+

=+=+

eKán 29ooooo29o

oooo

)60(cos1

29cos...3cos2cos1cos)60(cos89sin....63sin.62sin.61sinA ==

eday 2160cos o = dUcen¼ 5368709122A 29 == .

- 39 - PaK1

Page 47: HU NSEN 3 STR O N G...Problems and Solutions . GñkcUlrYmRtYtBinitübec© ... Mathematical Olympiad in China,Mathematical Olympiad Challenges, Mathematical Olympiad Treasures, International

KNitviTüaCMuvijBiPBelak

- 40 - PaK1

lMhat´TI16 eK[ CacMnYnBitEdlepÞógpÞat´lkçx&NÐ ½ z,y,x

0zcosycosxcos =++ nig cos 0z3cosy3cosx3 =++ cUrbgHajfa cos 0z2cosy2cosx2 ≤ . dMeNa¼Rsay bgHajfa cos 0z2cosy2cosx2 ≤ tamrUbmnþ cos acos3acos4a3 3 −= eKTaj )1(x3cosxcos3xcos4 3 +=

)3(z3coszcos3zcos4)2(y3cosycos3ycos4

3

3

+=

+=

bUkTMnak´TMng eKán ½ )3(,)2(,)1(

0zcos4ycos4xcos4 333 =++

¦ 0zcosycosxcos 333 =++ (eRBa¼ 0zcosycosxcos =++ nig 0z3cosy3cosx3cos =++ ) eKman 0zcosycosxcos =++ eKán zcosycosxcos −=+

Page 48: HU NSEN 3 STR O N G...Problems and Solutions . GñkcUlrYmRtYtBinitübec© ... Mathematical Olympiad in China,Mathematical Olympiad Challenges, Mathematical Olympiad Treasures, International

KNitviTüaCMuvijBiPBelak

elIkCaKUbeKán ½

zcosycosxcos3xcosycosxcoszcosycoszcosycosxcos3xcos

zcosycos)ycosx(cosycosxcos3xcoszcos)ycosx(cos

333

333

333

33

=++

−=+−

−=+++

−=+

eday 0zcosycosxcos 333 =++ eKTaj 0zcosycosxcos3 = naM[ ¦ 0xcos = 0ycos = ¦ 0zcos = . edaysn μtyk 0xcos = ena¼ zcosycos −= eKán ½

0)1zcos2(z2cosy2cosx2cos)1zcos2)(1ycos2)(1xcos2(z2cosy2cosx2cos

22

222

≤−−=

−−−=

dUcen¼bBaHaRtUvánRsaybBa¢ak´ .

- 41 - PaK1

Page 49: HU NSEN 3 STR O N G...Problems and Solutions . GñkcUlrYmRtYtBinitübec© ... Mathematical Olympiad in China,Mathematical Olympiad Challenges, Mathematical Olympiad Treasures, International

KNitviTüaCMuvijBiPBelak

- 42 - PaK1

lMhat´TI17 eK[RtIekaN mYy . bnÞatBu¼mMu kat´RCug ABC C,B,A

]AB[,]AC[,]BC[ erogKñaRtg´ . 'C,'B,'A

cUrRsayfa ½

0'CC

)2

BAsin(

'BB

)2

ACsin(

'AA

)2

CBsin(=

+

+

dMeNa¼Rsay

eyIgman S B'CC'ACCABC SS += eday Csinab

21SABC =

2Csin'CCb

21S 'ACC = nig

2Csin'CCa

21S B'CC =

eKán 2Csin'CC.a

21

2Csin'CC.b

21sibCab

21

+=

C

A 'C B

Page 50: HU NSEN 3 STR O N G...Problems and Solutions . GñkcUlrYmRtYtBinitübec© ... Mathematical Olympiad in China,Mathematical Olympiad Challenges, Mathematical Olympiad Treasures, International

KNitviTüaCMuvijBiPBelak

eKTaj )1(2Ccos

baab2

2Csin)ba(

Csinab'CC+

=+

=

eKman 2Ccos

2Csin2

2BAcos

2BAsin2

CsinBsinAsin

cba

+−

=−

=−

eday 2Csin

2C

2cos

2BAcos =⎟

⎠⎞

⎜⎝⎛ −π

=+

eKán 2Ccos2

BAsin

cba

=− naM[ )

2BAsin(.

bac

2Ccos −

−=

tam eKán )1( )2

BAsin(ba

abc2'CC 22

−−

=

eKTaj abc2

ba'CC

)2

BAsin( 22 −=

.

dUcKñaEdr abc2

ac'BB

)2

ACsin(;

abc2cb

'AA

)2

CBsin( 2222 −=

−−

=

dUcen¼ 0'CC

)2

BAsin(

'BB

)2

ACsin(

'AA

)2

CBsin(=

+

+

.

- 43 - PaK1

Page 51: HU NSEN 3 STR O N G...Problems and Solutions . GñkcUlrYmRtYtBinitübec© ... Mathematical Olympiad in China,Mathematical Olympiad Challenges, Mathematical Olympiad Treasures, International

KNitviTüaCMuvijBiPBelak

- 44 - PaK1

lMhat´TI18 eK[BhuFa P n)acosasinx()x( += Edl *INn∈ cUrrksMNlénviFIEckrvag nwg x)x(P 12 + .

dMeNa¼Rsay sMNl´énviFIEck tag )x(R CasMNl´énviFIEckrvag nwg )x(P 1x2 +

-ebI ena¼ 1n = acosasinx)x(P += dUcen¼ acosasinx)x(R += CasMnl´énviFIEck . -ebI eKán 2n ≥ )x(R)x(Q)1x()x(P 2 ++= Edl CaplEck nwg )x(Q BAx)x(R += eKán BAx)x(Q)1x()acosasinx( 2n +++=+ ebI ena¼ ix = BAi)acosasini( n +=+ ¦ A.iB)nasin(.i)nacos( +=+ eKTaj nig )nasin(A = )nacos(B = dUcen¼ )nacos()nasin(x)x(R += .

Page 52: HU NSEN 3 STR O N G...Problems and Solutions . GñkcUlrYmRtYtBinitübec© ... Mathematical Olympiad in China,Mathematical Olympiad Challenges, Mathematical Olympiad Treasures, International

KNitviTüaCMuvijBiPBelak

- 45 - PaK1

lMhat´TI19

eda¼RsayRbB&næsmIkar ⎩⎨⎧

−=−

+=+

byaxyxbayx

44

3

Edl nig b CacMnYnBitEdl aa 0b ≠+ .

dMeNa¼Rsay eda¼RsayRbB&næsmIkar

⎩⎨⎧

−=−

+=+

byaxyxbayx

44

3

smmUl 1y

yxbyax)yx(ba

44

3

⎩⎨⎧

−=−

+=+

443

44

3

yx)yx(y)yx(a

yxbyax)yx(ybyay

−++=+

+⎩⎨⎧

−=−

+=+

eday ena¼ 0ba ≠+ 0yx ≠+ eKTaj 23

443

xy3xyx

yx)yx(ya +=+

−++=

ehIy 323 yyx3a)yx(b +=−+=

eKánRbB&næ naM[ ⎩⎨⎧

=+

=+

byyx3axy3x

32

23

⎩⎨⎧

−=−

+=+3

3

bayxbayx

dUcen¼ 2

babay;2

babax . 3333 −−+

=−++

=

Page 53: HU NSEN 3 STR O N G...Problems and Solutions . GñkcUlrYmRtYtBinitübec© ... Mathematical Olympiad in China,Mathematical Olympiad Challenges, Mathematical Olympiad Treasures, International

KNitviTüaCMuvijBiPBelak

- 46 - PaK1

lMhat´TI20 cUrbgHajfa 9

)1nn)(1nn(19999

1n44 =⎟

⎞⎜⎝

⎛++++

∑=

dMeNa¼Rsay bgHajfa 9

)1nn)(1nn(19999

1n44 =⎟

⎞⎜⎝

⎛++++

∑=

eKman n1n)n1n)(n1n( 4444 −+=−+++

n1n1)n1n)(n1n( 4444

++=−+++

eKTaj 4444 n1n

)1nn)(1nn(1

−+=++++

9110000

)n1n()1nn)(1nn(

1

4

9999

1n

449999

1n44

=−=

−+=⎟⎠

⎞⎜⎝

⎛++++

∑∑==

dUcen¼ 9)1nn)(1nn(

19999

1n44 =⎟

⎞⎜⎝

⎛++++

∑=

.

Page 54: HU NSEN 3 STR O N G...Problems and Solutions . GñkcUlrYmRtYtBinitübec© ... Mathematical Olympiad in China,Mathematical Olympiad Challenges, Mathematical Olympiad Treasures, International

KNitviTüaCMuvijBiPBelak

- 47 - PaK1

lMhat´TI21 eK[GnuKmn_ kMnt´RKb´ f }0;1{IRx −−∈ eday ½

1x)x1(f)x(f)1x2(x +=++ .

cUrKNna )2009(f....)3(f)2(f)1(fS ++++=

dMeNa¼Rsay KNna )2009(f....)3(f)2(f)1(fS ++++= eKman )1(1x)

x1(f)x(f)1x2(x +=++

CMnYs eday xx1 kñúgsmIkar ( eKán ½ )1

)2(2xxx)x(f

2xx)

x1(f

1x1)x(f)

x1(f)1

x2(

x1

22

++

=+

+

+=++

dksmIkar nig Gg:nwgGg:eKán ½ )1( )2(

2x2x2)x(f

2x)1x(x2

2xxx1x)x(f

2xx)1x2(x

2

22

++

=++

++

−+=⎥⎦

⎤⎢⎣

⎡+

−+

eKTaján 1x

1x1

)1x(x1)x(f

+−=

+=

Page 55: HU NSEN 3 STR O N G...Problems and Solutions . GñkcUlrYmRtYtBinitübec© ... Mathematical Olympiad in China,Mathematical Olympiad Challenges, Mathematical Olympiad Treasures, International

KNitviTüaCMuvijBiPBelak

- 48 - PaK1

eKán [ ] ∑∑==

=−=⎟⎠⎞

⎜⎝⎛

+−==

2009

1k

2009

1k 20092008

200911

1k1

k1)k(fS

dUcen¼ 20092008)2009(f....)3(f)2(f)1(fS =++++= .

lMhat´TI22 cUrkMntRKb´KU éncMnYnKt´viC¢manebIeKdwgfa ½ )n;m(

)nm(13nm 22 +=+ .

dMeNa¼Rsay kMnt´RKb´KU ½ )n;m(

eKman )1()nm(13nm 22 +=+ -krNITI1 nm =

eKán naM[ n26n2 2 = 13n = dUcen¼ . 13nm ==

-krNITII2 nm ≠

eyIgBinitüeXIjfaebI ( CaKUcemøIyrbs ena¼eKán )n;m )1(

)m;n( k¾CaKUcemøIyrbs Edr . )1(

snμtfa m . tamvismPaB Cauchy Schwarz n<

Page 56: HU NSEN 3 STR O N G...Problems and Solutions . GñkcUlrYmRtYtBinitübec© ... Mathematical Olympiad in China,Mathematical Olympiad Challenges, Mathematical Olympiad Treasures, International

KNitviTüaCMuvijBiPBelak

eyIgman )nm(2)nm( 222 +<+

26nm)nm(26)nm( 2

<++<+

eday ena¼eKán nm < 26nmm2 <+< ¦ 13m <

eday ena¼eKTaj *INm∈ 12m1 ≤≤ . müa¨geTotsmIkar Gacsresr ½ )1(

)2(0m13mn13n 22 =−+− DIsRKImINg´ )m13m(4169 2 −−=Δ smIkar mancemøIykñúg kalNa )2( *IN Δ CakaerRákd éncMnYnKtviC©maness . eKyk INk)1k2()m13m(4169 22 ∈∀+=−− eKán )1k(k41)1k2()m13m(4168 22 +=−+=−− eKTaj )m13(m42)1k(k −+=+ eday 12m1 ≤≤ ena¼témøEdlGacrbs´plKuN )1k(k + KW ½

}84,82,78,72,64,54{)1k(k =+ . kñúgtémøTaMgRáMmYyen¼témøEdlCaplKuNcMnYnKt´tKñamanEt témø mYyKt´EdlRtUvnwg 9872 ×= }10;3{m = .

- 49 - PaK1

Page 57: HU NSEN 3 STR O N G...Problems and Solutions . GñkcUlrYmRtYtBinitübec© ... Mathematical Olympiad in China,Mathematical Olympiad Challenges, Mathematical Olympiad Treasures, International

KNitviTüaCMuvijBiPBelak

-cMeBa¼ eKán 3m = 0)2n)(15n(30n13n2 =+−=−− naM[ . 15n =

-cMeBa¼ eKán 10m = 0)2n)(15n(30n13n2 =+−=−− naM[ . 15n =

srubmkeKTTYlánKUcemøIy®áMKUKW ½ })10;15(;)3;15(;)13;13(;)15;10();15;3({)n;m( = .

- 50 - PaK1

Page 58: HU NSEN 3 STR O N G...Problems and Solutions . GñkcUlrYmRtYtBinitübec© ... Mathematical Olympiad in China,Mathematical Olympiad Challenges, Mathematical Olympiad Treasures, International

KNitviTüaCMuvijBiPBelak

- 51 - PaK1

lMhat´TI23 eK[RtIekaN mYyEkgRtg . eKyk ABC A ]AC[E∈

nig F Edl ]AB[∈ ABCAEF ∠=∠ nig ACBAFE ∠=∠ . eKtag nig F erogKñaCaeCIgcMeNalEkgén E nig F 'E '

elIRCug [ . ]BC

cUrRsayfa E BC'FFEFE' ≤++ ?

dMeNa¼Rsay Rsayfa E BC'FFEFE' ≤++ tag BC cAB,bAC,a === . RtIekaNEkg nig manmMu ABC AFE ABCAEF ∠=∠ vaCaRtIekaNdUcKña .

A

B C

F

E

'F 'E

Page 59: HU NSEN 3 STR O N G...Problems and Solutions . GñkcUlrYmRtYtBinitübec© ... Mathematical Olympiad in China,Mathematical Olympiad Challenges, Mathematical Olympiad Treasures, International

KNitviTüaCMuvijBiPBelak

eKánpleFobdMnUc kBCEF

ACAF

ABAE

=== Edl 0k > tag[pleFobdMnUc . eKTaj akEF,bkAF,ckAE === . RtIekaNEkg nig manmMu ABC C'EE ACB∠ rYmvaCaRtIekaN dUcKña . eKán

ECBC

'EEAB

= naM[ BC

EC.AB'EE = eday ckbAEACEC −=−= eKán

a)ckb(c'EE −

= . dUcKñaEdreKTaj

a)bkc(b'FF −

= eKán

a)bkc(bak

a)ckb(c'FFEFE'E −

++−

=++

abc2

a)cba(kbc2

akbbckakcbc

222

222

=−−+

=

−++−=

BIeRBa¼kñúgRtIekaNEkg eKman ABC 222 cba += (BItaK&r) tamvismPaB eKman GMAM − bc2cba 222 ≥+= eKTaj BCa

abc2

=≤ .dUcen¼ BC'FFEFE'E ≤++ .

- 52 - PaK1

Page 60: HU NSEN 3 STR O N G...Problems and Solutions . GñkcUlrYmRtYtBinitübec© ... Mathematical Olympiad in China,Mathematical Olympiad Challenges, Mathematical Olympiad Treasures, International

KNitviTüaCMuvijBiPBelak

- 53 - PaK1

lMhat´TI24 eK[ c,b,a CabIcMnYnBitviC¢man . cUrRsaybBa¢akfa ½

2)ba(4c

ba)ac(4b

ac)cb(4a

cb3 333 333 33

≤++

++

+++

+++

+

dMeNa¼Rsay Rsayfa ½ Rsayfa ½

2)ba(4c

ba)ac(4b

ac)cb(4a 3 +

cb3 333 3333

≤+

++

+++

++

+

)cb( 3

+

eKman b(bc3cb 33 )c+−+= PaB AM − an +

tamvism M eKmG bc2cb ≥+ eKTaj

2

2cbbc ⎟⎠⎞

⎜⎝⎛ +

≤ naM[ 3)cb(43)cb(bc3 +−≥+−

eKán 33333 )cb(41)cb(b

43)cb(c +=+−+≥+

aj eKT 3 33 )cb(4cb +≤+ ¦ 3 33 )cb(4acba + ++≤+ naM[ ( )1

cbacb

)cb(4a 3

cb33 ++

+≤

++

+

Page 61: HU NSEN 3 STR O N G...Problems and Solutions . GñkcUlrYmRtYtBinitübec© ... Mathematical Olympiad in China,Mathematical Olympiad Challenges, Mathematical Olympiad Treasures, International

KNitviTüaCMuvijBiPBelak

- 54 - PaK1

dUcKñaEdr ( )2cba

ac)ac(4b

ac3 33 ++

+≤

+++

nig ( )3c)ba(c

ba3 33 ba

ba4 ++

+≤

+

edaybUkTMnak´TMng eKán ½ +

+ .

)3(,)2(,)1(

2)ba(4c

ba)ac(4b

ac)cb(4a 3

cb3 333 3333

≤++

++

+++

++ .

++

lMhat´TI25 eK[sIVútcMnYn

−Bit

=⎩⎨⎧

≥∀+

==

−+ 1n,2a2aaa

1nn1n

10

2 ++

a1

cUrRsayfa a 1nn1naa ×= .

dMeNa¼Rsay a aa 2 +

Rsayf 1nn a +1n×=

a2a + −=

aa −−= RK ≥ −=

2aa2(c nn1nn1n

eKman a 1−

tag c b´ n

nn1n

1nnn

eKán c

1

n1n1n ++ aa

a) 2c +=−+−= −+

Page 62: HU NSEN 3 STR O N G...Problems and Solutions . GñkcUlrYmRtYtBinitübec© ... Mathematical Olympiad in China,Mathematical Olympiad Challenges, Mathematical Olympiad Treasures, International

KNitviTüaCMuvijBiPBelak

- 55 - PaK1

TMnak´TMngen¼bBa¢ak´fa CasIVútnBVnþmanplsgrYm )c( n 2d = 01nigtY 0c1 11aa =−=−= .

eKán 2n2)1n(2d)1n(cc 1n −=−=−+= 1nneKTaj a 2n2a −=− −

)

( ) (

)1n(n=1a

)12aa

1k2aa

n

0n

n

1k

n

1k1kk

−−

=−

−=− ∑∑==

2n(n −

eKTaj 1nna 2n +−= nig 1)1n()1n(a 2

1n ++−+=+ 2 ++= nig ¦ a 1n+ 1n21nn 1nna 24 ++=

+

nn(a −+eday )(1a 222221nn n)1n()1nn =+++−=+

¦ 1nnaa 241nn ++=+ .

dUcen¼

1nn1naaa 2 ++

×= .

Page 63: HU NSEN 3 STR O N G...Problems and Solutions . GñkcUlrYmRtYtBinitübec© ... Mathematical Olympiad in China,Mathematical Olympiad Challenges, Mathematical Olympiad Treasures, International

KNitviTüaCMuvijBiPBelak

- 56 - PaK1

lMhat´TI26 kñúgRtIekaN ABC mYycUrRsaybBa¢akfa ½

rR4111

≥ 2Csin

22Edl r nig

BsinA ++sin

R CakaMrgVg´carwkkñúg nig carwkeRkARtIekaN .

dMeNa¼

Rsay fa Rsay ( )1

r2Csin

2Bsin

2Asin

R4111++

tag ,bAC,aBCtag A,bAC,aBC

cB cBA ===

wsþIbTkUsIunUskñúgRtIekaN eKman ½ 222 − eday

tamRT ABC

Acos.bc2cba +=2Asin21Acos 2−=

ena¼ )2

sin21(bc2 − Acba 2222 −+=

eKTaj bc4

)cba)(cba()cb( 22 −−+−−bc4

a2Asin2 += =

tag 2

cbap =++ ( knø¼brimaRténRtIekaN )

eKán )cp(2cba −=−+ nig )bp(2cba −=+−

Page 64: HU NSEN 3 STR O N G...Problems and Solutions . GñkcUlrYmRtYtBinitübec© ... Mathematical Olympiad in China,Mathematical Olympiad Challenges, Mathematical Olympiad Treasures, International

KNitviTüaCMuvijBiPBelak

- 57 - PaK1

eKán bc

)cp)(bp(2Asin2 −−=

naM[ bc

)cp)(bp(2Asin −−= . dUcKñaEdreKTaj ½

ab)bp)(ap(

2Csin;

ac)cp)(ap(

2Bsin =

−−=

−−

eKán abc

)cp)(bp)(ap(2Cs sin

2Bsin

2Ain −−−

= eKman

R4abcpr =)cp)(bp)(ap(pS =−−−=

eKTaj S.R4ab nig c = S.rpS)cp)(bp)(ap( −

2

==−−

eKán R4r

S.R4S.r

2Csin

2Bsin

2Asin == .

vismPaB smmUleTAnwg ½ ( )1

( )22

2Csin

2Bsin

2Asin

2Bsin

2Asin

2Csin

2Asin

2Csin

2Bsin

2Csin

2Bsin

2Asin4

14

2Csin

1

2Bsin

1

2Asin

1

≥++

≥++

eday 2Asin

aap

bca)cp)(bp()si 2

−=

−−ap(2Csin

2Bn

2−=

Page 65: HU NSEN 3 STR O N G...Problems and Solutions . GñkcUlrYmRtYtBinitübec© ... Mathematical Olympiad in China,Mathematical Olympiad Challenges, Mathematical Olympiad Treasures, International

KNitviTüaCMuvijBiPBelak

- 58 - PaK1

eKTaj a

p −=

a

2Asin

2Csin

2Bsin

. dUcKñaEdreKTaján ½

bbp2

Asin2Csin −

2Bsin

= nig c

cp

2Csin

2Bs

2Asin in −

=

smmUleTAvismPaB ( )2 nwg ½ 2

ccp

bbp

aap

≥−

+−

+−

aB eKán ½ tamvismP GMAM −

a)ap(2a)ap(p −= −≥+ naM[ p

)ap(2a

ap −≥

dUcKñaEdr p

)bp(2b

bp −≥

− nig p

)cp(2c

cp −≥

− eKán

Bit2c

cpb

bpa

ap

)cp()bp()ap(2c

cpb

bpap

≥−

+−

+−

−+−+−≥

−+

−+

pa

dUcen¼r

4CsinBsin≥++

R

2

1

2

1

2Asin

. 1

Page 66: HU NSEN 3 STR O N G...Problems and Solutions . GñkcUlrYmRtYtBinitübec© ... Mathematical Olympiad in China,Mathematical Olympiad Challenges, Mathematical Olympiad Treasures, International

KNitviTüaCMuvijBiPBelak

- 59 - PaK1

lMhat´TI27 eK[ CabIcMnYnBitviC¢manEdl abc4c,b,a 1cba +++= UrbgHajfa ½ rbgHajfa ½ c

)cabcab(2c

bab

accb 222 ++

+a

222

++≥+

+

dMeNa¼Rsay Rsayfa ½

)ab(2 + cabcc

bab

aa

22

+≥+

++

aB eKman ½

ccb 2222 ++

tamvismP GMAM −

4 abcbaabc4 ++= naM[ 41c ≥+ eKTaj

1abc ≥

)1(abc31abc4cba ≥−=++ ( eRBa¼ ) 1abc ≥

tamvismPaB GMAM − eKán ½ ( )2

cab2ca2cccb 222 ++

bab2

cba

ba

a

222

++≥+

++ mPaB Cauchy - Schwarz eKman ½ tamvis

[ ]2222 )ca()bc()ab(3)cabcab( ++≤+ +

eday ])ab()ca()bc([abcca

=++2ab2

bca2 222 ++ bc2

Page 67: HU NSEN 3 STR O N G...Problems and Solutions . GñkcUlrYmRtYtBinitübec© ... Mathematical Olympiad in China,Mathematical Olympiad Challenges, Mathematical Olympiad Treasures, International

KNitviTüaCMuvijBiPBelak

- 60 - PaK1

eKTaj ( )3)cabcab(abc32

cab2

bca2

abc2 2++≥++

tam nig eKTaján½ )2( )3(

( )4)cabcab(abc32

cba

bac

acb 22

++ 2

2222

++≥+

++

22 +

eyIgman⎪⎩

⎪⎨

≥+

≥+

≥+

bca2)ab()ca(abc2)ca()bc(

cab2)bc()ab(

222

222

222

eKán )ab([2 2 )cba(abc2])ca()bc( +≥++ )cba(abc)ca()bc()ab( 222 ++≥++

aMgBIrnwg )()(ab(2EfmGg:T )caab(2)bc )ca)(bc(2++ a(abc3)cabc 2eKán bab( )c++≥++

≥++ ( tam eKman a abc3cb ( )1 )

naM[

eKTaj 2222 cba9)cabcab( ≥++

¦ abc3cabcab ≥++ ( )5 )cabcab(2)cabcab(

abc32 2 ++≥++

eKTaján ½ tam )4( nig )5(

)cabcab(2c

baaa

cb 22222

≥+

+++

bc2

+++ .

Page 68: HU NSEN 3 STR O N G...Problems and Solutions . GñkcUlrYmRtYtBinitübec© ... Mathematical Olympiad in China,Mathematical Olympiad Challenges, Mathematical Olympiad Treasures, International

KNitviTüaCMuvijBiPBelak

- 61 - PaK1

lMhat´TI28 eK[sIVút a{ ½ }n kMnteday

= nig 1a0 4a....a.aa n101n +=+ cMeBa¼RKb´ . 1n ≥

cUrbgHajfa 2aa 1nn =− + cMeBa¼RKb´ . 1n ≥

dMeNa¼Rsay bgHajfa 2aa 1nn =− +

g;teXIjfaRKb´ eyIgse INk∈ eKman . eKman

n 0ak > . eKman

0ak >

4a....a.aa n101n 4a....a.aa n101n +=+ 1n102neKán a 4a.....a.a += ++

2n102n

n0n102n

)2a.....aa(

4)a....a.4)a.....aa(

4)4a......a)(a.....a.a(

+=

++= 1

a(a

aa

2n102n

n10

a

++=

+

+

+

eKTaj 2a... n..aaa 102n +=+ 2)4a(a 1n2n +−= ++ ¦ 2aa 2n1n =− ++

dUcen¼ 2aan − 1n =+ .

Page 69: HU NSEN 3 STR O N G...Problems and Solutions . GñkcUlrYmRtYtBinitübec© ... Mathematical Olympiad in China,Mathematical Olympiad Challenges, Mathematical Olympiad Treasures, International

KNitviTüaCMuvijBiPBelak

- 62 - PaK1

lMhat´TI29 eK[ CabIcMnYnBitviC¢manEdl c,b,a 1cabcab =++ . cUrbgHajfa 16)

a1c()

c1b()

b1a( 222 ≥+++++

¼

dMeNa Rsay a bgHajf 16)

a1c()

c1b()

b1a( 222 ≥+++++

tag 222 )a1c()

c1b()

b1a(A = +++++

⎟⎠⎞

⎜⎝⎛ ++= a ++++++

ac

cb

ba2

c1

b1

a1cb 222

222

eyIgBinitü 22 acabcab

a1 ++= 2a

bcac

ab

++= 22

cab1

bbcab ++

= 2bac

bc

ba+= +

22 ccabcab

c1 ++= 2c

abca

cb

++= PaB eKman ½ tamvism GMAM −

3cab

bca

abc,

ab≥+ 2

ac

ca,2

bc

cb;2ba

222 ≥++≥+≥+ eKán 93222

c1

b1

a1

2 + 22 =+++≥+

Page 70: HU NSEN 3 STR O N G...Problems and Solutions . GñkcUlrYmRtYtBinitübec© ... Mathematical Olympiad in China,Mathematical Olympiad Challenges, Mathematical Olympiad Treasures, International

KNitviTüaCMuvijBiPBelak

- 63 - PaK1

ehIy 3a

müa¨geT c2

c.cb.

ba3

ac

cb

ba

3 =≥++ ot b,ab2ba 22222 ≥ ca2ac;bc2 +≥+≥+

222naM[ a2 ca2bc2ab2c2b2 ++≥++ aM[ n 1cabcabcba 222 =++≥++

eKán 1A 16)3(29 =+ .

+≥

dUcen¼ 16)acb

1c()1b()1a( 222 ≥+++++ .

lMhat´TI30 eK[

432 2cos2222,

2cos222,

2c22 os π

=++π

=+π IcUrrkrUbmnþTUeTA nig RsaybBa¢ak´

a¼pg .

=

BI«TahrN_xagelrUbmnþen

dMeNa¼Rsay

eKman rkrUbmnþTUeTA ½

,2

cos222,2

cos22 32

π=+

π=

42cos2222 π

=++

Page 71: HU NSEN 3 STR O N G...Problems and Solutions . GñkcUlrYmRtYtBinitübec© ... Mathematical Olympiad in China,Mathematical Olympiad Challenges, Mathematical Olympiad Treasures, International

KNitviTüaCMuvijBiPBelak

- 64 - PaK1

tamlMnaM«TahrN UeTAdUcxa ½_eyIgGacTajrkrUbmnþT geRkam

1n

)n(2

co222 s2.........2 +

π=+ +++ .

eyIgtagRsaybBaØak´rUbmnþen¼ ½

)n(

n 2......222A ++++= RK b´ n∈ *IN

eyIgman 21 2cos22A π

== Bit favaBitdl´tYTI W ½ eyIg«bma Kp

1p

)p(

p 2222A2

cos2...... +

π=++ Bit

ayfavaBitdl´tYTI

++=

eyIgnwgRs 1p + KW 2p1p 2cos2A ++

π= Bit

eyIgman p1p A2A +=+ aytamkar«bma ed 1pp 2

cos2A +

π=

eyIgán 2p2p

21p1pA

2cos2

2cos4

2cos22 ++++

π=

π=

π+= Bit

dUcen¼ 1ncos22.........222 +

π=++++

)n(2

.

Page 72: HU NSEN 3 STR O N G...Problems and Solutions . GñkcUlrYmRtYtBinitübec© ... Mathematical Olympiad in China,Mathematical Olympiad Challenges, Mathematical Olympiad Treasures, International

KNitviTüaCMuvijBiPBelak

- 65 - PaK1

lMhat´TI31 edayeFIVvicartamkMeN acMnYn ½ IncUrRsaybBa¢akf

2 + ckdacnwg Ct´Fm

1n1n 9A ++= E anic©cMeBa¼RKb´ n6 11

cMnYnK μCati n .

dMeNa¼Rsay RsaybBa¢ak´facMnYn 1n6A 1n

n 92 ++ += Eckdacn wg11

-ebI 0n = eKán 1192A0 =+= Bit ( Eckdac´nwg11 ) -ebI eKán 091n = 1911292A 27

1 ×==+= Bit wg ).

favaBitdltYTI KW ( Eckdacn 11

eyIg«bma p 1p1p6p 92A ++ +=

eyIgnwgRsayfavaBitdl YTI KW Ec

p 9

Eckdac´nwg ´t11 1P +

2p7p61p 92A ++

+ += kdacnwg11 2peyIgman 1 2A 7p6 ++

+ += 11p1p66

1p+ )9.29(A p62p)92(2 +++

+ −++=

9.55A64

9A64

1p

p

A

)649(A+

p1p+

1pp1

++

−=

−+=

Page 73: HU NSEN 3 STR O N G...Problems and Solutions . GñkcUlrYmRtYtBinitübec© ... Mathematical Olympiad in China,Mathematical Olympiad Challenges, Mathematical Olympiad Treasures, International

KNitviTüaCMuvijBiPBelak

- 66 - PaK1

edaytamkar«bma Eckdac wg ehIy .

án p 9.55A64A

pA ´n 11 1p9.55 +

Eckdac´nwg11

eKTaj 1pp1

++ −= Eckdacnwg it .

nA

11 BdUcen¼cMnYn n1n6 92 1++ += kdacnwg11 MeBa¼ Ec Canic©cb´cMnYnKt´Fm μCatin .

RK

lMhat´TI32 97831 nn

neK[ INn,617043E nn 0 ∈−+−=

ac ic©R

cUrbgHajfa nE Eckd ´nwg 187 Can Kb´ INn∈ .

dMeNa¼Rsay a Eckdac´nwg Canic b´ bgHajf ©RKnE 187 INn∈ ½

× Edl nig CacMnYnbfmrvagyfa Eckdac´nwg eyIgEtUvRsay

[eXIjfa Eckdac´ nig . ab 1nn −=

eday 187 = Kña . 1711 11 17

dUcen¼edIm,IRsa nE 187

nE 11 17

eKman ba)(ba(a 2n1nn N.)ba()b....+++−=− −−−

Page 74: HU NSEN 3 STR O N G...Problems and Solutions . GñkcUlrYmRtYtBinitübec© ... Mathematical Olympiad in China,Mathematical Olympiad Challenges, Mathematical Olympiad Treasures, International

KNitviTüaCMuvijBiPBelak

- 67 - PaK1

Edl 1n aaN 1n2n b.......b −−− +++= án 610709743831E nnnn

n ∈eK , INn−+−=

( ) ( )

*INq,p;)q9p8(11q99p88q)610709(p)743831(

610709743831 nnnn

∈+=+=−+−=−+−=

709743831E nnnnn

eKTaján Eckdacnwg 11 . nE

müa¨geToteKman½ INn,610 ∈−+−=

*IN;;)213(1734221

)743709()610831()743709()610831( nnnn

∈βαβ−α=β−α=β−+α−=

−+−=

Taján Eckdacnwg . ´nwg Canic©RKb´

eK nE 17

dUcen¼ n EckdacE 187 INn∈ .

Page 75: HU NSEN 3 STR O N G...Problems and Solutions . GñkcUlrYmRtYtBinitübec© ... Mathematical Olympiad in China,Mathematical Olympiad Challenges, Mathematical Olympiad Treasures, International

KNitviTüaCMuvijBiPBelak

- 68 - PaK1

lMhat´TI33 cUrRsaybBa¢ak´fa

1n21n

1.....3

112

1 +<+

++++

dMeN Rsay ybBa¢ak´fa ½ Rsa

1n21n

1.....3

12

11 < ++

+++ man

+

eyIg

1k21

1k1kk1kk1k 11

+=

+++>

++=−+

¦ k1k1k2

1−+<

+

ebI 1222

1:1k = −<

ebI 2332

1:2k = −<

ebI 3442

1:3k −<= ------------------------

ebI ---------

n1n1n2

1:nk −+<+

=

Page 76: HU NSEN 3 STR O N G...Problems and Solutions . GñkcUlrYmRtYtBinitübec© ... Mathematical Olympiad in China,Mathematical Olympiad Challenges, Mathematical Olympiad Treasures, International

KNitviTüaCMuvijBiPBelak

- 69 - PaK1

eKán 1121

−n)1n

1.........3

12

1( +<+

+++

Taj

eK 21n21n

1........3

12

1−+<

++++

1n2

11n21n

1.......32

+

1

<

−+<+

+++

dUcen¼

11+

1n21n

1.....3

112

1 +<+

++++ . lMhat´TI34 eK[ n cMnYnBitviC¢man . cUrRsaybBa¢akfa ½

n321 a.........;;a;a;a

nn321 ............aa ++ 21n3 a....a.a.anaa ≥++

dMeNa¼Rsay RsaybBa¢ak´fa ½

nn321n321 a.a.ana..........aaa ≥++++

=

a...... -cMeBa¼ eKman 2n 2121 aa2aa ≥+

Ul smm 0)aa( 221 ≥− Bit

Page 77: HU NSEN 3 STR O N G...Problems and Solutions . GñkcUlrYmRtYtBinitübec© ... Mathematical Olympiad in China,Mathematical Olympiad Challenges, Mathematical Olympiad Treasures, International

KNitviTüaCMuvijBiPBelak

- 70 - PaK1

-«bmafavaBitdltYTI KW ½ k

kk321k321 aaa ++ a......a.a.aka.......... ≥++ Bit

eyIgnwgRsayfavaBitdl´tYTI 1k + KW ½ 1k

1kk3211kk32 a.a......a.a.a)1k(aa..........aaa +++ +≥+++++ 1

tag *INk,0a,0x;

x)a......aaax(

)x(f k

1kk321 ∈≥>∀

+++++=

+ eyIgán ½

2

kk

xax(x)a(

)x('f1k

k2121 )a.....a....aax)(1k +++++−+++++

=

[ ]

( )2

k21k

k21

2k21

kk21

xa....aakx)a....aax(

x)a....aax(x)1k()a....aax(

−−−−++++=

++++−+++++=

ebI naM[ 0)x('f =k

a.....aax k21 +++=

cMeBa¼ k

a..... kaax 21 ++ eKán ½

+=

kk211kk21

k2

ka +1

1kk

k21

k21

ka.....aa

)1k()k

a......aa(f

a....a

)....k

a....aa(

)k

aa(f

⎟⎠⎞

⎜⎝⎛ +++

+=+++

++

+21 aaaa.... ++++++

=++

+

+

+

Page 78: HU NSEN 3 STR O N G...Problems and Solutions . GñkcUlrYmRtYtBinitübec© ... Mathematical Olympiad in China,Mathematical Olympiad Challenges, Mathematical Olympiad Treasures, International

KNitviTüaCMuvijBiPBelak

- 71 - PaK1

taragGefrPaBén x

)a.....aax()x(f

kk21 ++++

= x o

k..aa 21 a. k+++ ∞+

)x('f −.

+.

)x(f

m

PaBxagelIcMeBa¼ tamtaragGefr 0x ≥∀ eyIgTaján ½

kk3211k

1kk21

ka.....aaa)1k(

x)a......aa

x()x(f

⎠⎞

⎜⎝⎛ ++++

+

++++

+

+

ma ≥

=

tamkar«b kk321k321 a......a.a.aka..........aaa ≥++++

smmUl k21

ka.....++ k21 a.........a.ak

aa≥⎟

⎠⎞

⎜⎝⎛ +

eKTaj k211k

1kk21 a......aa)1k(

x)a.......aax( +

+

+≥++++

naM[ (*)xa.......a.a)1k(a.....aax 1kk21k21

++≥++++ yk CYskñúg 0ax 1k >= + ( )*

Page 79: HU NSEN 3 STR O N G...Problems and Solutions . GñkcUlrYmRtYtBinitübec© ... Mathematical Olympiad in China,Mathematical Olympiad Challenges, Mathematical Olympiad Treasures, International

KNitviTüaCMuvijBiPBelak

- 72 - PaK1

eKán 1k

1kk3211kk321 aaa +++ a.a......a.a.a)1k(aa.......... +++ +≥++ Bit

dUcen¼ nn321n321 .....aaa +++ a......a.a.ana..... ≥+ .

lMhat´TI35 eda¼RsayRbB&næsmIkar ½

⎩⎨

ytanxtan3

=+

=++

34ytanxtan28ytanytanxtan6xtan

3

4224

Edl 2

yx0 π<<< .

dMeNa¼Rsay eda¼RsayRbB&næsmIkar ½

⎩⎨

+ tanxtantanxt 33

=

=++

34yyan28ytanytanxtan6xtan 4224

smmUl⎩⎨⎧

=+

=++

)tanxtan4y

2(316ytanxtan4

)1(28ytanytanxtan6xtan33

4224

Page 80: HU NSEN 3 STR O N G...Problems and Solutions . GñkcUlrYmRtYtBinitübec© ... Mathematical Olympiad in China,Mathematical Olympiad Challenges, Mathematical Olympiad Treasures, International

KNitviTüaCMuvijBiPBelak

- 73 - PaK1

bUksmIkar nig eKán ½ )1( )2(

424 4(3162)tax(tan ==+ )31()328yn +=++ eday

2yx0 π<<< ena¼eKán ytanxtan0 <<

eKTaj )3(31ytanxtan +=+ eFIVplsg 1( nig 2() eKán ½ )

424 )31()324(1628x(tan −−− 3)ytan −=== eKTaj )4(1tanxtan −=− eRB3y a¼ ytanxtan0 <<

ar nig án bUksmIk )4( eK tan2)3( 2x = eKTaj 1xtan = naM[

4x π= .

IVplsgsmIkar nig eKán eF )3( )4( 32ytan2 = Taj eK 3ytan = naM[

3y π= .

cen¼eKánKUcemøIy dU )3

y,4

x( π=

π= .

Page 81: HU NSEN 3 STR O N G...Problems and Solutions . GñkcUlrYmRtYtBinitübec© ... Mathematical Olympiad in China,Mathematical Olympiad Challenges, Mathematical Olympiad Treasures, International

KNitviTüaCMuvijBiPBelak

- 74 - PaK1

lMhat´TI36 eK[ CaRCugrbs´RtIekaNmYynigtag

nRtIekaN . a

c,b,a p

Caknø¼brimaRtécUrbgHajf ⎟

⎠⎜⎛ ++≥

−+

−+

− c1

b1

a12

cp1

bp1

ap1

dMeNa¼⎝

Rsay ajfa bgH ⎟

⎠⎞

⎜⎝⎛ ++≥

−+

−+

− c1

b1

a12

cp1

bp1

ap1 MnYnBit > eKman ½

2

cMeBa¼RKb´c > 0B;0A

0A)BA( 22 ≥BB.A2 +−2 ≥++

n

=− ¦ A2 B.A4BB.A2

B.A4)BA( 2 ≥+

eKTajáBA

4B.ABA

+≥

+ ¦ )1(BA

4B11

A +≥+

yk B,apA bp −=−= án eK cbap2BA =−−=+ CYskñúg )1(

eKán )2(c4

bp11

≥−

+

dUcKñaEdreKTaj½ ap −

.

Page 82: HU NSEN 3 STR O N G...Problems and Solutions . GñkcUlrYmRtYtBinitübec© ... Mathematical Olympiad in China,Mathematical Olympiad Challenges, Mathematical Olympiad Treasures, International

KNitviTüaCMuvijBiPBelak

- 75 - PaK1

)3(a4

cp1

bp1

≥−

+−

nig )4(b4

ap1

cp1

≥−

+−

aB Gg:nwgGg:eKán ½ B Gg:nwgGg:eKán ½ bUkvismP )4(,)3(,)2( )4(,)3(,)2(

c4

b4

a4

cp2

bp2

ap2

++≥−

+−

+−

dUcen¼ ⎟⎠⎞

⎜⎝⎛ ++≥

−+

−+

−p1

c1

b1

a12

cp1

bp1

a .

lMhat´TI37 k¿cUrRsaybBa¢ak´fa ½

3n41n4

1n41n4

1n43n4

+−

<+−

<+− cMeBa¼RKb´

¿TajbgHajfa ½ *INn∈

x

3951n2 ×+

n43

)1n4(.......13)1n4(.......1173

+<

+×××−×××× . 1

<

dMeNa¼Rsay k¿RsaybBa¢akfa

3n41n4

1n41n4

1n43n4

+−

<+−

<+−

tamvismPaB eKGacsresr ½ GMAM −

Page 83: HU NSEN 3 STR O N G...Problems and Solutions . GñkcUlrYmRtYtBinitübec© ... Mathematical Olympiad in China,Mathematical Olympiad Challenges, Mathematical Olympiad Treasures, International

KNitviTüaCMuvijBiPBelak

)3n4)(1n4(1

1n41

)3n4)(1n4(1n4

)3n4)(1n4(22n8

)3n4)(1n4(2)3n4()1n4(

+−<

+

+−>+

+−>+

+−>++−

KuNGg:TaMgBIrnwg )1n4( − eyIgán ½ )1(

3n41n4

1n41n4

+−

<+−

müa¨geTot )1n4)(3n4(2)1n4()3n4( +−>++−

)1n4)(3n4(1n4

)1n4)(3n4(22n8

+−>−

+−>−

EckTaMgBIrnwg eyIgán ½ )1n4( +

)2(1n43n4

1n41n4

+−

>+−

tamTMnak´TMng nig eKTaján ½ )1( )2(

3n41n4

1n41n4

1n43n4

+−

<+−

<+− .

x¿TajbgHajfa½

3n43

)1n4(.......1395)1n4(.......1173

1n21

+<

+××××−××××

<+

- 76 - PaK1

Page 84: HU NSEN 3 STR O N G...Problems and Solutions . GñkcUlrYmRtYtBinitübec© ... Mathematical Olympiad in China,Mathematical Olympiad Challenges, Mathematical Olympiad Treasures, International

KNitviTüaCMuvijBiPBelak

tamsRmayxagelIeyIgman 3n41n4

1n41n4

1n43n4

+−

<+−

<+−

cMeBa¼ eyIgán ½ n;.....;3;2;1n =

3n41n4

1n41n4

1n43n4

1511

1311

139

117

97

95

73

53

51

+−

<+−

<+−

−−−−−−−−−−−−−−−−−−

<<

<<

<<

KuNvismPaBTaMgen¼Gg:nwgGg:eKán ½

3n43

1n41n4.........

1311.

97.

53

1n41

+<

+−

<+

edayeKman 1n2

14n4

11n4

1+

=+

>+

. dUcen¼

3n43

)1n4(.......1395)1n4(.......1173

1n21

+<

+××××−××××

<+

.

- 77 - PaK1

Page 85: HU NSEN 3 STR O N G...Problems and Solutions . GñkcUlrYmRtYtBinitübec© ... Mathematical Olympiad in China,Mathematical Olympiad Challenges, Mathematical Olympiad Treasures, International

KNitviTüaCMuvijBiPBelak

lMhat´TI38 eK[ a CabIcMnYnBitviC¢man . c;b;

cUrbgHajfa 23

bac

acb

cba

≥+

++

++

dMeNa¼Rsay bgHajfa

23

bac

acb

cba

≥+

++

++

tag ⎪⎩

⎪⎨

=+=+=+

pbanacmcb

eKán ( ) ( ) ( ) pnmbaaccb ++=+++++ naM[

2pnmcba ++

=++

eKTaj

⎪⎪⎪

⎪⎪⎪

−+=

+−=

−+=

2pnmc

2pnmb

2mpna

eKán

- 78 - PaK1

Page 86: HU NSEN 3 STR O N G...Problems and Solutions . GñkcUlrYmRtYtBinitübec© ... Mathematical Olympiad in China,Mathematical Olympiad Challenges, Mathematical Olympiad Treasures, International

KNitviTüaCMuvijBiPBelak

- 79 - PaK1

p2pnm

n2pnm

m2mpn

bac

acb

cba −+

++−

+−+

=+

++

++

⎥⎦

⎤⎢⎣

⎡−⎟

⎞⎜⎝

⎛++⎟

⎞⎜⎝

⎛+⎟

⎞⎜⎝⎛ ++

++

+ nm

mn

acb

cba

+⎠

=+

3np

pn

pm

mp

21

bac

amvismPaB eKán ½ t GMAM −

2np

pn;2

pm

mp;2

nm

mn+ ≥+≥+≥

eKán ( )23322

21

bac

acb

ca

=−+≥ 2b

++

++

++

dUcen¼ 23

bac

acb

cba

≥+

++

+ . +

lMhat´TI39 k¿KNna

n)1n(3.21

2.11

−++ Edl 2n > 1....

4.31

++ x¿edayeRbIvismPaB GMAM − én n( )1− cMnYnxageRkam ½

n)1n(1;

4.33.22.11;1;1

− cUrbgHajfa 2n )!n(n < .

dMeNa¼Rsay k¿KNna ∑

=⎥⎦

⎤⎢⎣

⎡−

=−

++++n

2k k)1k(1

n)1n(1....

4.31

3.21

2.11

Page 87: HU NSEN 3 STR O N G...Problems and Solutions . GñkcUlrYmRtYtBinitübec© ... Mathematical Olympiad in China,Mathematical Olympiad Challenges, Mathematical Olympiad Treasures, International

KNitviTüaCMuvijBiPBelak

- 80 - PaK1

eKman k1

1k1

k)1k()1k(k

k)1k(1

−−

=−

−−=

eKán n11

k1

1k1

k)1k(1 n

2k

n

k∑=2

−=⎟⎠⎞

⎜⎝⎛ −

−=⎥

⎤⎢⎣

⎡−

∑=

dUcen¼ n11

n)1n(1...

2.11+ .

.31

3.21

−=−

+++4

. n(n <

tamvismPaB ½ x¿ bgHajfa 2)!n

GMAM −

nn321n321 a......a.a.ana.....aaa ++ ≥++

eKán ½ 0a; k ≥∀

)1n(

n)1n(1......

3.21.

2.11)1n(

n)1n(1....

3.21

2.11

−−>

−+++

2n2

1n

)1n(2

)1n(

)1n(

)!n(n1

>⎟⎠⎞

⎜⎝⎛ n

)!n(n

)!n(n

n1

!)1n(!n1)1n(

n1n

!)1n(!n1)1n(

n11

<⇒

>

−−>

−−>−

dUcen¼ 2n )!n(n < .

Page 88: HU NSEN 3 STR O N G...Problems and Solutions . GñkcUlrYmRtYtBinitübec© ... Mathematical Olympiad in China,Mathematical Olympiad Challenges, Mathematical Olympiad Treasures, International

KNitviTüaCMuvijBiPBelak

- 81 - PaK1

lMhat´TI40 eK[ ;b;a c CabIcMnYnBitxusBIsUnü .

UrbgHajfa cac

cb

ba

ac

cb

ba 222

++≥⎟⎠⎞

⎜⎝⎛+⎟

⎠⎞

⎜⎝⎛+⎟

⎠⎞

⎜⎝

dMeNa¼Rsay

bgHajfa ac

cb

ba

ac

ba

+≥⎟⎠⎞

⎜⎝⎛+⎜

⎝⎛

cb 222

+⎟⎠⎞

⎜⎝⎛+⎟

⎠⎞

tamvismPaB eKman ½ GMAM −

3b

⎜⎝

+⎟⎠

⎜⎝

ac.

cb.

ba3

ac

cba

32

2

2

2

2

2222

=≥⎟⎠⎞

⎜⎝⎛+⎟

⎠⎞⎛⎞⎛

eKTaj )1(3ac

cb

ba 222

≥⎟⎠⎞

⎜⎝⎛+⎟

⎠⎞

⎜⎝⎛+⎟

⎠⎞

⎜⎝⎛

müa¨geToteKman

⎪⎪⎪⎪

⎪⎪⎪⎪

≥+⎟⎠⎞

⎜⎝⎛

≥+⎟⎠⎞

⎜⎝⎛

≥+⎟⎠⎞

⎜⎝⎛

ac21

ac

cb21

cb

ba21

ba

2

2

2

eKTaján )2(ac

cb

ba23

ac

cb

⎜⎝⎛ +≥+⎟

⎠⎞

⎜⎝⎛+⎜

⎝⎛+⎜

⎝⎛

ba 222

⎟⎠⎞+⎟

⎠⎞

⎟⎠⎞

Page 89: HU NSEN 3 STR O N G...Problems and Solutions . GñkcUlrYmRtYtBinitübec© ... Mathematical Olympiad in China,Mathematical Olympiad Challenges, Mathematical Olympiad Treasures, International

KNitviTüaCMuvijBiPBelak

- 82 - PaK1

bUkvismPaB nig Gg: nig Gg: eKán ½ )1( )2(

⎟⎠⎞

⎜⎝⎛ +++≥⎥

⎤⎢⎣

⎡⎟⎠⎞

⎜⎝⎛+⎟

⎠⎞

⎜⎝⎛+⎟

⎠⎞

⎜⎝⎛+

ac

cb

ba23

ac

cb

ba23

222

⎟⎠⎣ ⎝ b⎞

⎜⎝⎛ ++≥⎥

⎤⎢⎡

⎟⎠⎞

⎜⎝⎛+⎟

⎠⎞

⎜⎝⎛+⎟

⎠⎞

⎜⎛

ac

cb

ba2

ac

cba2

222

Ucen¼ dac

cb

ba

ac

cb

b⎝a 222

++≥⎟⎠⎞

⎜⎝⎛+⎟

⎠⎞

⎜⎝⎛+⎟

⎠⎞

⎜⎛ .

lMhat´TI41 eK[ ;y;x z CabIcMnYnBitviC¢manminsUnü . UrbgHajfa c 222222 zyx

9z1

y1

x

1++

≥++

dMeNa¼Rsay bgHajfa 222222 yxzyx +

≥++z

9111+

vismPaBen¼GacsresreTACa ½

9z

zyxy

zyxx

zyx2

222

2

222

2

222

≥++

+++

+++

6zy

yz

zx

xz

yx

xy

9zy

zx1

yz

yx1

xz

xy1

2

2

2

2

2

2

2

2

2

2

2

2

2

2

2

2

2

2

2

2

2

2

2

2

≥+++++

≥++++++++

Page 90: HU NSEN 3 STR O N G...Problems and Solutions . GñkcUlrYmRtYtBinitübec© ... Mathematical Olympiad in China,Mathematical Olympiad Challenges, Mathematical Olympiad Treasures, International

KNitviTüaCMuvijBiPBelak

eday 2yx.

xy2

yx

xy

2

2

2

2

2

2

2

2

=≥+

2zx.

xz2

zx

xz

2

2

2

2

2

2

2

2

=≥+ nig 2yz.

zy2

yz

zy

2

2

2

2

2

2

2

2

=≥+

eKán 6222zy

yz

zx

xz

yx

xy

2

2

2

2

2

2

2

2

2

2

2

2

=++≥+++++ Bit

dUcen¼ 222222 zyx9

z1

y1

x1

++≥++ .

lMhat´TI42 eKman xlog1

1

aay −= nig ylog11

aaz −=

cUrRsayfa zlog11

aax −= .

dMeNa¼Rsay Rsayfa zlog1

1

aax −=

eKman xlog11

aay −= naM[ xlog1

1yloga

a −=

eKTaj )1(ylog

11xloga

a −=

- 83 - PaK1

Page 91: HU NSEN 3 STR O N G...Problems and Solutions . GñkcUlrYmRtYtBinitübec© ... Mathematical Olympiad in China,Mathematical Olympiad Challenges, Mathematical Olympiad Treasures, International

KNitviTüaCMuvijBiPBelak

dUcKñaEdr ylog11

aaz −= naM[ ylog1

1zloga

a −=

eKTaj )2(zlog

11yloga

a −=

ykTMnak´TMng eTACYskñúg eKán ½ )2( )1(

zlog11

11xlog

a

a

−−=

1)z(logzlog

1xloga

aa −

−=

zlog11xlog

1)z(logzlog1zlog

xlog

aa

a

aaa

−=

−−−

=

eKTaján zlog11

aax −= .

- 84 - PaK1

Page 92: HU NSEN 3 STR O N G...Problems and Solutions . GñkcUlrYmRtYtBinitübec© ... Mathematical Olympiad in China,Mathematical Olympiad Challenges, Mathematical Olympiad Treasures, International

KNitviTüaCMuvijBiPBelak

- 85 - PaK1

lMhat´TI43

eKBinitüsIVúténcMnYnBit ( kMnt´eday )un⎪

⎪⎨

−−

=

=

+ 1u24u5

u

4u

n

n1n

1

cUrKNna u CaGnuKmn_én . n n

dMeNa¼Rsay KNna u CaGnuKmn_én ½ n n

smIkarsMKal´énsIVút 1u24u5

un

n1n −

−=+ KW

1r24r5r

−−

=

¦ man¦s 04r6r2 2 =+− 2acr;1r 21 ===

tagsIVút 2u1uv

n

nn −

−=

eKán nn

n

n

n

n

n

1n

1n1n v3)

2u1u

(32

1u24u5

11u24u5

2u1u

v =−−

=−

−−

−−−

=−−

=+

++

TMnak´TMngen¼bBa¢ak´fa CasIVútFrNImaRtmanersug )v( n

nigtY 3q =23

2u1u

v1

11 =

−−

= .

Page 93: HU NSEN 3 STR O N G...Problems and Solutions . GñkcUlrYmRtYtBinitübec© ... Mathematical Olympiad in China,Mathematical Olympiad Challenges, Mathematical Olympiad Treasures, International

KNitviTüaCMuvijBiPBelak

tamrUbmnþ 233

23qvv

n1n1n

1n =×=×= −−

eday 2u1uv

n

nn −

−= eKTaj

123

131v1v2u n

n

n

nn

−=

−−

=

dUcen¼ 23

)13(2v n

n

n −−

= . lMhat´TI44

eKBinitüsIVúténcMnYnBit kMnt´eday )u( n⎪⎩

⎪⎨

−−

=

=

+ 1u4u3

u

3u

n

n1n

1

cUrKNna CaGnuKmn_én . nu n

dMeNa¼Rsay KNna CaGnuKmn_én ½ nu n

smIkarsMKal´énsIVút 1u4u3

un

n1n −

−=+ KW

1r4r3r

−−

=

¦ 0)2r(4r4r 22 =−=+− eKTaj¦s 2rr 21 == . tagsIVút

2u1v

nn −=

- 86 - PaK1

Page 94: HU NSEN 3 STR O N G...Problems and Solutions . GñkcUlrYmRtYtBinitübec© ... Mathematical Olympiad in China,Mathematical Olympiad Challenges, Mathematical Olympiad Treasures, International

KNitviTüaCMuvijBiPBelak

- 87 - PaK1

eKán 2u

1v1n

1n −=

++ eday

1u4u3

un

n1n −

−=+

eKán 2u1u

2u24u31u

vn

n

nn

n1n −

−=

+−−−

=+

eKman 12u

12u1u

vvnn

nn1n =

−−

−−

=−+ efr

naM[ CasIVútnBVnþmanplsgrYm )v( n 1d = nigtY 1

231

2u1v

11 =

−=

−= .

tamrUbmnþ n1n1d)1n(vv 1n =−+=−+= eday

2u1v

nn −= eKTaj

n1n2

n12

v12u

nn

+=+=+=

dUcen¼ n

1n2un+

= .

Page 95: HU NSEN 3 STR O N G...Problems and Solutions . GñkcUlrYmRtYtBinitübec© ... Mathematical Olympiad in China,Mathematical Olympiad Challenges, Mathematical Olympiad Treasures, International

KNitviTüaCMuvijBiPBelak

- 88 - PaK1

lMhat´TI45 eKBinitüsIVúténcMnYnBit kMnt´eday ½ )u( n

⎩⎨⎧ = 4u1

∈=+ *INn,uu n1n

cUrKNna CaGnuKmn_é .

MeNa¼

nu n n

d Rsay aGnuKmn_én KNna nu C n

eKman n1 uu = n+

eKán n1n ulnln21u =+

gen¼bB¢ak´fa (l n CasIVútFrNImaRtmanersug TMnak´TMn un )21q =

tamrUbmnþ ( ) 1n2

1

1n1

n 4ln4ln1ulnuln −=== − n1 2

q× −

dUcen¼ ( ) 1n2

1

n 4u −= .

Page 96: HU NSEN 3 STR O N G...Problems and Solutions . GñkcUlrYmRtYtBinitübec© ... Mathematical Olympiad in China,Mathematical Olympiad Challenges, Mathematical Olympiad Treasures, International

KNitviTüaCMuvijBiPBelak

- 89 - PaK1

lMhat´TI46 eK[sIVúténcMnYnBit kMnt´eday ½ )u( n

5u0 = nig 5nu21u n ++ 1n =+ ( INn∈ )

cUrKN CaGnuKmn_én

MeNa¼

na nu énsIVút n . )u( n

d Rsay sIVút CaGnuKmn_én

u

KNna nu én )u( n n

eyIgtag bvnn an ++= Edl IRb,a ∈ eyIgán baanvu 1n1n +++= ++ eday 5nu

21u n1n+ ++=

eKán 5n)banvn ++v(21baan n1 ++=++++

(*))b21a5(n)1a

21(v

21v n1n ⎥⎦

⎤⎢⎣⎡ −−++−+=+

ebI ⎪⎪⎩

⎪⎪⎨

⎧ −+− 01a1

=−− 0b21a5

2 naM[ 3b;2a ==

TMnak´TMng eTACa ( )* n1n v21v =+ naM[ CasIVútFrNImaRt. )v( n

manersug 21

= . q

Page 97: HU NSEN 3 STR O N G...Problems and Solutions . GñkcUlrYmRtYtBinitübec© ... Mathematical Olympiad in China,Mathematical Olympiad Challenges, Mathematical Olympiad Treasures, International

KNitviTüaCMuvijBiPBelak

- 90 - PaK1

tamrUbmnþ 0 qv × nn v= eday 6n2vbanvu nnn ++=++= eKán 0cMeBa¼ n = 6v00 u +=

u0naM[ 56v 0 16 −=−=−= eKán nnn 2

1211v −=×−= .

dUcen¼ 6n221u nn ++−= .

lMhat´TI47 edayeRbIGnumanrYmKNitviTüacUrKNnalImItxageRkam ½

2x2x22.....222

limLn 2x −−++++++

= →

( man n ra¨DIkal´ ) .

MeNa¼

d Rsay ageRkam ½ KNnalImItx

eyIgman 2x

2x22.....222limLn 2x −

−+++++→

+=

eyIgán

Page 98: HU NSEN 3 STR O N G...Problems and Solutions . GñkcUlrYmRtYtBinitübec© ... Mathematical Olympiad in China,Mathematical Olympiad Challenges, Mathematical Olympiad Treasures, International

KNitviTüaCMuvijBiPBelak

- 91 - PaK1

41

2x21lim

)2x2)(2x(4)x2(lim

2x2x2limL

2x1 = → 2x2x=

++=

++−−+

=−

−+→→

212x2x2

2x2x2

41L

41

2x2x2lim

2x221limL

)2x22)(2x(2x2lim

2x2x22limL

==−−+

×+++

=

+++−

−+=

−−++

=

→→

→→

eyIgsnμtfavaBitdllMdab´TI KW k kk 41L =

´TeyIgnwgRsayfavaBitdl´lMdab I )1k( + KW 1k1k 41L ++ =

eyIgman 2x

2x2.....222 2limL

2x1k −−++

→+ knigPaKEbgnwg

++++=

KuNPaKy2x2..2222)x(M ++++= . +++

eKán )x(M).2x(

4x2....2222limL2x1k −

−++++++=

→+

Bit1kkkk1k

2x2x1k

41

41

41L

41L

)2(M1L

2x2x2....222lim

)x(M1limL

++

→→+

=×==×=

−−+++++

×=

dUcen¼ n2xn 41

2x2x22.....222

limL =−

−++++++=

Page 99: HU NSEN 3 STR O N G...Problems and Solutions . GñkcUlrYmRtYtBinitübec© ... Mathematical Olympiad in China,Mathematical Olympiad Challenges, Mathematical Olympiad Treasures, International

KNitviTüaCMuvijBiPBelak

- 92 - PaK1

lMhat´TI48 eK[sIVúténcMnYnBit kMnt´elI eday ½ )U( n IN

⎪⎩

⎪⎨⎧ = 2U0

∈+=+ INn,U2U n1n

¿cUrKNna CaGnuKmn_én . KuN 1 U....

k nnU

x¿KNnapl 20n UUUP n××××= .

eNa¼

dM Rsay aGnuKmn_én ½

KNna nU C n

eyIgman4

cos220U π==

8

cos24

cos22U2U 01π

+=+=

itdl´tYTI KW «bmafavaB p 2pp 2cos2U +

π=

I eyIgnwgRsayfavaBitdl´tYT )1p( + KW 3p1p+ 2cos2U +

π= Bit

eyIgman p1p U2U +=+ Et r«bmtamka a 2pp 2cos2U +

π=

eyIgán 3p3p

22p cos4U ++

π=

π 1p 2cos2

22cos22 ++

π=+= Bit

Page 100: HU NSEN 3 STR O N G...Problems and Solutions . GñkcUlrYmRtYtBinitübec© ... Mathematical Olympiad in China,Mathematical Olympiad Challenges, Mathematical Olympiad Treasures, International

KNitviTüaCMuvijBiPBelak

- 93 - PaK1

dUcen¼ 2nn 2cos2U +

π= .

aplKuN 0n U....UUPx¿ KNn 1U n2 ××××= naM[ tamrUbmnþ 2a2sin = acosasin

ana2n

sisiacos2 =

2n2n

n 1k2sin +

0k

n

0k2k

n

0k2kkn

2sin

1

2sin

2sin

)

2sin

()2

cos2()U(P

++

= =+

=+

π=

π

π

=

π

π

==∏ ∏∏

dUcen¼ 2n

n

2sin

1P+

π= .

Mhat´TI49 l

MnYnBit kMnt´elI eday ½ eK[sIVúténc )U( n IN

22U0 = nig INn,

211

U2

n1n ∈∀

−−+

U=

KNna nuKmn_én .

n CaGU n

Page 101: HU NSEN 3 STR O N G...Problems and Solutions . GñkcUlrYmRtYtBinitübec© ... Mathematical Olympiad in China,Mathematical Olympiad Challenges, Mathematical Olympiad Treasures, International

KNitviTüaCMuvijBiPBelak

- 94 - PaK1

dMeNa¼Rsay KNna CaGnuKmn_én ½ n

U n

eyIgman4

sin20 ==2 π U

8

sin2

4sin11

2U11

U1 =

22

0 π=

π−−

=−−

favaBitdl´tYTI KW «bma p 2pp 2sinU +

π=

I eyIgnwgRsayfavaBitdl´tYT )1p( + KW 3p1pU + 2sin += Bit π

eyIgman 2

U11U 1p

−−=+

2p kar«bma 2+pp 2

sinU π=

eyIgán 2

2U2p

2+

sin111p+

π−−

=

3p

3+=

p2

2p

2sin

22

sin2

22

cos1+

+ π=

π

=

π−

Bit

dUcen¼ 2nn 2sinU +

π= .

Page 102: HU NSEN 3 STR O N G...Problems and Solutions . GñkcUlrYmRtYtBinitübec© ... Mathematical Olympiad in China,Mathematical Olympiad Challenges, Mathematical Olympiad Treasures, International

KNitviTüaCMuvijBiPBelak

- 95 - PaK1

lMhat´TI50 eK[sIVúténcMnYnBit )u( n kMnt´eday½

⎧ = au0

⎩⎨

>∈−=+ 2a,IN,2uu 2n1n ∀n

rRsaybBa¢ak´fa cUnn 2

22

2

n 24aa

24aau ⎟

⎟⎠

⎞⎜⎜⎝

⎛ −−+⎟

⎟⎠

⎞⎜⎜⎝

⎛ −+=

dMeNa¼Rsay

RsaybBa¢ak´fa nn 2

22

2

n 24aa

24aau ⎟

⎟⎠

⎞⎜⎜⎝

⎛ −−+⎟

⎟⎠

⎞⎜⎜⎝

⎛ −+=

ebI 0n = eKán a2

4aa2

4aau22

0 =−−

+−+

= Bit

avaBit KW

«bm dltYTI kkk 2

22

2

k 24⎟⎟⎠

aa2

4aau⎞

⎜⎜⎝

⎛ −−+⎟

⎟⎠

⎞⎜⎜⎝

⎛ −+=

itdl´tYTeyIgnwgRsayfavaB I 1k + KW 1k1k 2

22

2

1k 22u +⎟

⎟⎠

⎜⎜⎝

=+

4aa4aa++

⎟⎟⎠

⎞⎜⎜⎝

⎛ −−⎞⎛ −+

eyIgman 2uu 2k1k −=+

Page 103: HU NSEN 3 STR O N G...Problems and Solutions . GñkcUlrYmRtYtBinitübec© ... Mathematical Olympiad in China,Mathematical Olympiad Challenges, Mathematical Olympiad Treasures, International

KNitviTüaCMuvijBiPBelak

- 96 - PaK1

Ettamkar«bma kk 2

22

2

k 24aa

24aau ⎟

⎟⎠

⎞⎜⎜⎝

⎛ −−+⎟

⎟⎠

⎞⎜⎜⎝

⎛ −+=

eyIgán 22

4aa2

4aau

22

22

2

1k

kk

−⎥⎥

⎢⎢

⎟⎟⎠

⎞⎜⎜⎝

⎛ −−+⎟

⎟⎠

⎞⎜⎜⎝

⎛ −+=+

24

4aa22

4aa2

4aau22

22

22

1k

1k1k

−+−

×+⎟⎟⎠

⎞⎜⎜⎝

⎛ −−+⎟

⎟⎠

⎞⎜⎜⎝

⎛ −+=

++

+

1k1k 2

22

2

1k 24aa

24aau

++

⎟⎟⎠

⎞⎜⎜⎝

⎛ −−+⎟

⎟⎠

⎞⎜⎜⎝

⎛ −+=+ Bit

dUcen¼ nn 2

22

2

n 24aa

24aau ⎟

⎟⎠

⎞⎜⎜⎝

⎛ −−+⎟

⎟⎠

⎞⎜⎜⎝

⎛ −+= .

Page 104: HU NSEN 3 STR O N G...Problems and Solutions . GñkcUlrYmRtYtBinitübec© ... Mathematical Olympiad in China,Mathematical Olympiad Challenges, Mathematical Olympiad Treasures, International

KNitviTüaCMuvijBiPBelak

- 97 - PaK1

lMhat´TI51 k¿cUrkMnt´elxénGBaØat éncMnYn d,c,b,a abcd ebIeKdwgfa ½

dcba9abcd =× x¿cMeBa¼témø ,b,a EdlánrkeXIjxagelIcUrbBa¢ak´fa d,c

cMnYn abcd nig dcba suTæEtCakaer®ákd .

dMeNa¼Rsay k¿kMntelxénGBaØat ½ d,c,b,a

eKman )1(dcba9abcd =× k´TMng eKTajánté EtmYyKtKW . tamTMna mø )1( a 1a =

cMeBa¼ 1a = eKán )2(1dcb9bcd1 =× tamTMna g )2( eKk´TMn Taján 9d = eRBa¼ =×

es d 819

manelxxagcug μI 1 . cMeBa¼ 9d = eKán )3(1cb999bc1 × =

k´TMn Taján tamTMna g )3( eK 0b = inG T ) ( eRBa¼ 9b× m acmanRtaTuke

Page 105: HU NSEN 3 STR O N G...Problems and Solutions . GñkcUlrYmRtYtBinitübec© ... Mathematical Olympiad in China,Mathematical Olympiad Challenges, Mathematical Olympiad Treasures, International

KNitviTüaCMuvijBiPBelak

- 98 - PaK1

cMeBa¼ b eKán 0= )3(01c999c10 =× k´TMn Taján tamTMna g )3( eK 8c = eRBa¼ 72989c =×=×

Efm 8 [elxxagcugesμI 0 . cMeBa¼ 8c = eKán 91089 9801=× .

adUcen¼ 0b,1 9d,8c, === .= ´facMnYn x¿bBa¢ak abcd nig dcba suT aer®ákd ½

0

æEtCakcMeBa¼ 98c,b,1a d, ==== eKán ½

2331089abcd = = nig 2991980dcba == suTæEtCakaer®ákd

.

Page 106: HU NSEN 3 STR O N G...Problems and Solutions . GñkcUlrYmRtYtBinitübec© ... Mathematical Olympiad in China,Mathematical Olympiad Challenges, Mathematical Olympiad Treasures, International

KNitviTüaCMuvijBiPBelak

- 99 - PaK1

lMhat´TI52 sn μtfa HUNSEN CacMnYnmYymanelxRáMmYyxÞg´Edl

eBlEdleKKuNcMnYn YlTæplCacMnYnmanelx N,E,S,N,U,H Caelx . en¼nwg 3 eKTT

RáMmYyxÞg´KW STRONG Edl G,N,O,R,T,S Caelx cUrrkcMnYn HU ig NSEN n STRONG

= 8NcMeBa¼ N = nigcMeBa¼ S,3S,6 9== . usKñatageday ). (Gkßrx elxxusKña

STRONG3

HUNSEN×

dMeNa¼Rsay rkcMnYn HUNSEN nig STRONG

Page 107: HU NSEN 3 STR O N G...Problems and Solutions . GñkcUlrYmRtYtBinitübec© ... Mathematical Olympiad in China,Mathematical Olympiad Challenges, Mathematical Olympiad Treasures, International

KNitviTüaCMuvijBiPBelak

- 100 - PaK1

eKman ½

STRONG

3HU×

NSEN

k¿cMeBa¼ 3S,6N ==

G6TRO33

6E63HU×

tamtaragRbmaNviFIKuNen¼bBa¢ak´fa 1H =

cMeBa¼ 1H = taragviFIKuNeTACa ½

G6TRO3

36E36U1

×

tamtaragviFIKuNen¼eKTaján 8G = eRBa¼

eTACa ½

36×

manelxagcugesμI 8 . cMeBa¼ 8G = taragviFIKuN

68TRO3

36E63U1

×

tamtaragviFIKuNen¼eKTaján 5E = eRBa¼ Efm 3E× 1

Page 108: HU NSEN 3 STR O N G...Problems and Solutions . GñkcUlrYmRtYtBinitübec© ... Mathematical Olympiad in China,Mathematical Olympiad Challenges, Mathematical Olympiad Treasures, International

KNitviTüaCMuvijBiPBelak

- 101 - PaK1

manelxcugeRkayesμInwg 6 . cMeBa¼ 5E = taragviFIKuN TACa e ½

68TRO3

363U1

×56

tamtaragviFIKuNen¼eKTaján 0O = eRBa¼ Efm

½

33× 1

manelxcugeRkayesμInwg 0 . cMeBa¼ 0O = taragviFIKuNeTACa

068TR33

6U1 356×

tamtaragviFIKuNen¼eKTaján 9R = eRBa¼ Efm 36× 1

manelxcugeRkay esμInwg 9 . cMeBa¼ 9R = taragviFIKuNeTACa ½

9068T33

6U1 356×

Page 109: HU NSEN 3 STR O N G...Problems and Solutions . GñkcUlrYmRtYtBinitübec© ... Mathematical Olympiad in China,Mathematical Olympiad Challenges, Mathematical Olympiad Treasures, International

KNitviTüaCMuvijBiPBelak

- 102 - PaK1

tamtaragen¼bBa¢ak´fa minGacFMCag eTehIyedayGkßr xusKñatagedayelxxusKñaena¼eKTaján

U 2

2U = EtmYyKt BIeRBa¼ ehIy minGacyktémø ¦ 1H,0O == U 0 1

dEdleTotáneT . cMeBa¼ 2U = taragviFIKuNeTACa ½

90T3 68

56

tamtaragviFIKuNen¼eKTaján

3× 1263

7T = BIeRBa¼fa ½

379068 3

126356×

edayeRbobeFobtaragviFIKuN ½

STRONG nwg 3

HUNSEN×

379068

3126356×

Page 110: HU NSEN 3 STR O N G...Problems and Solutions . GñkcUlrYmRtYtBinitübec© ... Mathematical Olympiad in China,Mathematical Olympiad Challenges, Mathematical Olympiad Treasures, International

KNitviTüaCMuvijBiPBelak

- 103 - PaK1

eKTaján ½ 126356 nigHUNSEN = 379068STRONG =

CacMnYnEdlRtUvrk . ¿cMeBa¼ x 9S,8N ==

tamrebobdUcxagelIeKán ½ eyIgeda¼Rsay

9567843×

318928

edayeRbobeFobtaragviFIKuN ½

STRONG3

HUNSEN× nwg

9567843

318928×

eKTaján ½ 318928HUNSEN = nig 956784STRONG =

CacMnYnEdlRtUvr

k .

Page 111: HU NSEN 3 STR O N G...Problems and Solutions . GñkcUlrYmRtYtBinitübec© ... Mathematical Olympiad in China,Mathematical Olympiad Challenges, Mathematical Olympiad Treasures, International

KNitviTüaCMuvijBiPBelak

- 104 - PaK1

lMhat´TI53 cUrbMeBjRbmaNviFIbUkxageRkam ½

YENOM

EROM

dMeNa¼

DNES+

(GkßrxusKñatagedayelxxusKña )

Rsay bMeBjRbmaN UkxageRkam ½ viFIb

YENOMEROM

+DES

N

cemøIyrbs´taragviFIbUken¼KW ½

256015801

7659+

.

( sisßeda¼RsayedayxøÜnÉg )

Page 112: HU NSEN 3 STR O N G...Problems and Solutions . GñkcUlrYmRtYtBinitübec© ... Mathematical Olympiad in China,Mathematical Olympiad Challenges, Mathematical Olympiad Treasures, International

KNitviTüaCMuvijBiPBelak

- 105 - PaK1

lMhat´TI54 eK[ACacMnYnmYymanelxRáMxÞg´EdlelxxÞg´vaerobtamlMdab´

x;1x;x +x;2x;1 ++ ehIy CacMnYnmYymanelx ´

B

RáMmYyxÞg´EdlelxxÞg´vaerobtamlMdabx;1x;1x;2x;1x;x −++ .

Ha

dMeNa¼

+

cUrbg jfaebI A CakaerRákdena¼ B k¾Cakaer®ákdEdr .

Rsay

eyIgmanelxTaMgRáMxÞg´én A erobtamlMdab´ karbgHaj ½

x;1x2x;1x;x +++ naM[ 92x1x0 ≤+<; x +<< ¦ 7x0 ≤< .

a¼ -ebI 1x = en 211112321A == kd .

-ebI ena¼ A = mnCakaer®ákd . I e minEmnCakaer®ákd .

= minEmnCakaer®ákd .

-ebI 2x = ena¼ 23432A = minEmnCakaer®á3x = 34543 minE

-eb na¼ 4x = 45654A =

-ebI x ena¼ 5 56765A =

Page 113: HU NSEN 3 STR O N G...Problems and Solutions . GñkcUlrYmRtYtBinitübec© ... Mathematical Olympiad in China,Mathematical Olympiad Challenges, Mathematical Olympiad Treasures, International

KNitviTüaCMuvijBiPBelak

- 106 - PaK1

-ebI ena¼ minEmnCakaer®ákd . 6x = 67876A =

-ebI ena¼ minEmnCakaer®ákd . 7x = 78987A =

srubmkeKán x øEdl[ A Cakaer®ákd 1= Catém . ´vaerobtaml

;1

eday elxxÞg Mdab´ ½ B

x;1;2xx;x x;1x −++

MeBa¼ eKán +

ena¼c 1x = 2351123201B == Cakaer®ákd . ¾Cakaer®ákdEdr . dUcen¼ ebI A CakaerRákdena¼ B k

lMhat´TI55 eK[cMnYn

)elxn2(

11.....11A 11= nig )elxn(

444.....444B =

cUrbgHajfa 1B ++ CakaerRákd . A

dMeNa¼Rsay bgHajfa A 1B ++ CakaerRákd ½ eyIgman

9110111A

n2 −=.....111

n2(

=)elx

Page 114: HU NSEN 3 STR O N G...Problems and Solutions . GñkcUlrYmRtYtBinitübec© ... Mathematical Olympiad in China,Mathematical Olympiad Challenges, Mathematical Olympiad Treasures, International

KNitviTüaCMuvijBiPBelak

- 107 - PaK1

9

)110(4444.....444n

n(

−=B =

)e

lx

eyIgán 19

)19

1BAn

+−

+=++

10(4110 n2 −

2n

nn2

nn2

32101BA

9410.4101BA

99410.41101BA

⎟⎟⎠

⎞⎜⎜⎝

⎛ +=++

++=++

+−+−=++

dUcen¼ CakaerRákd . 1BA ++

lMhat´TI56 cMnYnmYymanelxbYnxÞg´EdlelxxÞg´vaerobtamlMdab´

b;b;a;a . IeKdwgfavaCakaerRákd . rkcMnYnena¼eb

dMeNa¼Rsay rkcMnYnEdlCakaerRákd ½ tag N CacMnYnEdlRtUvrk

Page 115: HU NSEN 3 STR O N G...Problems and Solutions . GñkcUlrYmRtYtBinitübec© ... Mathematical Olympiad in China,Mathematical Olympiad Challenges, Mathematical Olympiad Treasures, International

KNitviTüaCMuvijBiPBelak

- 108 - PaK1

eyIgán bb10a100a1000aabbN +++==

++=

)ba100(11Nb11a1100N

+= [ ] )1()ba(a9911+=

eday 9a0 ≤< nig b0 9≤≤ ena¼ 18ba0 ≤+< Mng edIm,I[ GacCakaerRákdlu¼RtaEt huKuNén ehIy

tamTMnak´T )1( N

ba + CaB 11 18ba0 ≤+< ena¼eKRtUv[ Etm .

Ga11ba =+ YyKt

TMnak´TMng )1( csresr ( ) 1a9(1111a9911 2 )N +=+= ehtue k a n¼ N Cakaer®á dkalN 1a9 + Cakeday ≤

aerRákd . 9a ≤ naM[ 101 811a9 ≤+≤ eKTaj ½

)

= (K ñúg )

161a9 =+ (Kμan¦skñúg IN

1a9 =+ (K μan¦skñúg IN ) 25

1a9 + μan¦sk36 IN

(K μan¦skñúg ) 491a9 =+ IN

naM[ 7a641a9 =+ = e Iy h 4711b =−=

cen¼cMnYnEd KW 4

(K μan¦skñúg I ) . 811a9 =+ N

dU lRtUvkMnt´ena¼ 774 288= .

Page 116: HU NSEN 3 STR O N G...Problems and Solutions . GñkcUlrYmRtYtBinitübec© ... Mathematical Olympiad in China,Mathematical Olympiad Challenges, Mathematical Olympiad Treasures, International

KNitviTüaCMuvijBiPBelak

- 109 - PaK1

lMhat´TI57 eK 0889333,1089 2man 33 22 = 111088893333,11==

. agelIcUrrkrUbmnþTUeTA nigRsaybBa¢ak´rUbmnþen¼pg

1111088889333332 =

BI«TahrN_x

dMeNa¼Rsay eKman 111088893333,110889333,108933 222 === .

IgGacbeg;ItrUbmnþTUeTAdUcxageRkam ½ ...

)1)1n()

2

−−

=

1111088889333332 =

tamlMnaMen¼ey9888.....8880111.....111333..333

n(n(

.

eyIgtag karRsaybBa¢ak´rUbmnþ ½

9888.........8880111..........111A)1n()1n( −−

=

Page 117: HU NSEN 3 STR O N G...Problems and Solutions . GñkcUlrYmRtYtBinitübec© ... Mathematical Olympiad in China,Mathematical Olympiad Challenges, Mathematical Olympiad Treasures, International

KNitviTüaCMuvijBiPBelak

)n(

2

2n2nnn2

n1nn2

1n1n1n

)1n(

1n

)1n(

333............333

3110

9)110(

9110.210

9818010.81010

910)110(9810)110(

91

910.888.........88810111.........111

=

⎟⎟⎠

⎞⎜⎜⎝

⎛ −=

−=

+−=

+−+−=

+−+−=

++×=

+

−+−

+

dUcen¼eKánrUbmnþ ½ 9888.....8880111.....111333.....333

)1n()1n()n(

2

−−

= .

- 110 - PaK1

Page 118: HU NSEN 3 STR O N G...Problems and Solutions . GñkcUlrYmRtYtBinitübec© ... Mathematical Olympiad in China,Mathematical Olympiad Challenges, Mathematical Olympiad Treasures, International

KNitviTüaCMuvijBiPBelak

lMhat´TI58 eKman 999800019999,998001999,980199 222 === . 9999800001999992 =

BI«TahrN_xagelIcUrrkrUbmnþTUeTAnigRsaybBa¢ak´rUbmnþen¼pg

dMeNa¼Rsay tambMrab´eKman ½

999800019999998001999

980199

2

2

2

=

=

=

9999800001999992 = tamlMnaMen¼eyIgGacbeg;ItrUbmnþTUeTAdUcxageRkam ½

1000.....0008999.....999999.....999)1n()1n()n(

2

−−

= .

karRsaybBa¢ak´rUbmnþ ½ eyIgtag 1000.........0008999..........999A

)1n()1n( −−

=

- 111 - PaK1

Page 119: HU NSEN 3 STR O N G...Problems and Solutions . GñkcUlrYmRtYtBinitübec© ... Mathematical Olympiad in China,Mathematical Olympiad Challenges, Mathematical Olympiad Treasures, International

KNitviTüaCMuvijBiPBelak

)n(

22n

nn2

n1nn2

n1n1n

n1n

)1n(

999............999)110(110.210

110.81010110.810)110(

110.810999.........999

=−=

+−=

++−=

++−=

++×=

+

+−

+

dUcen¼eKánrUbmnþ½

1000.....0008999.....999999.....999)1n()1n()n(

2

−−

= .

lMhat´TI59 eKman 444355566666,443556666,435666 222 === . 4444355556666662 =

BI«TahrN_xagelIcUrrkrUbmnþTUeTAnigRsaybBa¢ak´rUbmnþen¼pg

dMeNa¼Rsay tambMrab´eKman

444355566666,443556666,435666 222 === 4444355556666662 = .

- 112 - PaK1

Page 120: HU NSEN 3 STR O N G...Problems and Solutions . GñkcUlrYmRtYtBinitübec© ... Mathematical Olympiad in China,Mathematical Olympiad Challenges, Mathematical Olympiad Treasures, International

KNitviTüaCMuvijBiPBelak

- 113 - PaK1

tamlMnaMen¼eyIgGacbeg;ItrUbmnþTUeTAdUcxageRkam ½ 6555.....5553444.....444666.....666

)1n()1n()n(

2

−−

= .

karRsaybBa¢ak´rUbmnþ ½ eyIgtag ½

6555.........5553444..........444A)1n()1n( −−

=

)n(

2

2

)n(

2

)n(

2nnn2

nn2

nn1nn2

1nn1n1n

)1n(

n1n

)1n(

666............666)999.........999(32

)999.........999(94)110(

94

9410.810.4

94)52740(1010.4

9545010.510.2710.410.4

610)110(9510.310)110(

94

610555.......55510.310444........444

=⎥⎥⎦

⎢⎢⎣

⎡=

=−=+−

=

+−−−=

+−++−=

+−++−=

+×++×=

+

−+−

+

dUcen¼eKánrUbmnþ½ 6555.....5553444.....444666.....666

)1n()1n()n(

2

−−

=

Page 121: HU NSEN 3 STR O N G...Problems and Solutions . GñkcUlrYmRtYtBinitübec© ... Mathematical Olympiad in China,Mathematical Olympiad Challenges, Mathematical Olympiad Treasures, International

KNitviTüaCMuvijBiPBelak

- 114 - PaK1

lMhat´TI60 eKman

111111445556,11114556,1156 22 2222 =−=− .

BI«TahrN_xagelIcUrrkrUbm nigRsaybBa¢ak´rUbmnþen¼pg

=−

1111111144455556 22 =−

nþTUeTA

dMeNa¼Rsay rkrUbmnþTUeTA nigRsaybBa¢ak´ ½

22

2

=−

=−

1111111144455556 =− tamlMnaMEdleK[eyIgGacsresrrUbmnþTUeTAdUcxageRkam ½

)n(

nigRsaybBa¢ak´ ½

11111144555611114556

11

22

2

=−

=−

1111111144455556 22 =− tamlMnaMEdleK[eyIgGacsresrrUbmnþTUeTAdUcxageRkam ½

)n2()n()n(

eKman eKman

11111144555611114556

11562

22 =− 562

22 =−

22

22 111.......111445.....444556....555 =− 22 111.......111445.....444556....555 =−)n2()n(

sRmayepÞógpÞat´rUbmnþen¼ ½

)n(

2445...tag )n(

2n ..444556....555T −=

Page 122: HU NSEN 3 STR O N G...Problems and Solutions . GñkcUlrYmRtYtBinitübec© ... Mathematical Olympiad in China,Mathematical Olympiad Challenges, Mathematical Olympiad Treasures, International

KNitviTüaCMuvijBiPBelak

- 115 - PaK1

( )

( )9

999.........999

9110)

9110(110

1)110(941)110(

951)110(

941)110(

95

1)110(941)110(

95

)1444......444(1555.....555=

)n2(n2n

n

nnnn

2n

2n

22

=−

=−

+=

⎥⎦⎤

⎢⎣⎡ −−−+−⎥⎦

⎤⎢⎣⎡ +−++−=

⎟⎠⎞

⎜⎝⎛ +−−⎟

⎠⎞

⎜⎝⎛ +−=

+−+

Bit .

dUcen¼ )n2()n(

2

)n(

111.......111445.....5....555 =

)n2(n 111..........111T =

2 44456 − .

lMhat´TI61 eK[ 011nn aa..........aaA = − nig 011nn a2a.........aaB ×−= −

fa Eckdac´nwg t Eckdacnwg . Rsay lu¼RtaE

A 7 B 7

dMeNa¼Rsay ´nwg ena¼naM[man ebI B Eckdac 7 INq∈ Edl q7B =

eKán q7a2a........aa 011nn =×−− .. naM[ )1(a a2q7a..........a 011nn ×+=−

Page 123: HU NSEN 3 STR O N G...Problems and Solutions . GñkcUlrYmRtYtBinitübec© ... Mathematical Olympiad in China,Mathematical Olympiad Challenges, Mathematical Olympiad Treasures, International

KNitviTüaCMuvijBiPBelak

- 116 - PaK1

eyIgman )2(a10a...........aaaa.........aaA 1nn= − 011nn01 +×= −

yk CUskñúg eKán q7

)1( )2(

00 a)a2(A 10+×+= )a3q10(7a21q07aa20q70A 0000 +=+=++=

naM[ Eckdacnwg . ¼RtaEt Eckdac´nwg .

A 7

dUcen¼ Eckdacnwg luA 7 B 7

lMhat´TI62 eK[cMnYn 0121nn aaa......aaA −=

a.....,,a, Caelx . yfacMnYn Eck alNa

Edl 0 a,a n21

cUrRsa dac´nwg 6 kA

0n21 a)a..aa(4y . ++

dMeNa¼

++= Eckdacnwg 6 .

Rsay eyIgman 0121nn aaa......aaA = −

n2

10 a10.....a10a10aA 2n++++=

Page 124: HU NSEN 3 STR O N G...Problems and Solutions . GñkcUlrYmRtYtBinitübec© ... Mathematical Olympiad in China,Mathematical Olympiad Challenges, Mathematical Olympiad Treasures, International

KNitviTüaCMuvijBiPBelak

- 117 - PaK1

eyIgman )6(mod410 ≡

eyIgán )6(mod410

)6(mod410)6(mod410

n

3

2

−−−−−−−−−−≡

)6(mod)a.....aa(4aA n210 ++++≡ Eckdac´nwg lu¼RtaEt

n1

edIm,I[ A 6

(mod0)a....aa(4a 20 )6≡++++ cen¼cMnYn Eckdacnwg kalNa

2

dU A 6

n1 a)a...aa(4y 0++++= Eckdacnwg 6 . lMhat´TI63 cUrkMntelx a nig edIm,I[cMnYn b abba CaKUbéncMnYnKt´ .

dMeNa¼

Rsay nig kMnt´elx a b

tag ab10b100a1000abbaN +++== b110a1001

+=+=

NN

)b10a91(11

Page 125: HU NSEN 3 STR O N G...Problems and Solutions . GñkcUlrYmRtYtBinitübec© ... Mathematical Olympiad in China,Mathematical Olympiad Challenges, Mathematical Olympiad Treasures, International

KNitviTüaCMuvijBiPBelak

- 118 - PaK1

edIm,I[ N CaKUbéncMnYnKt´lu¼RtaEt *INk,k121k1110a91 332b ∈==+

eday 9b0,9a0 ≤≤≤< ena¼ 909b10a910 ≤+< eKán 909k1210 3 ≤<

smmUl121627 +

121909k0 3 =≤< naM[ 1k =

eKTaján 121b10a91 =+ naM[ 10

a9112b 1−=

ena¼ eday 0b ≥ 0≥10

a91121− ¦

91301a +≤

91121

= naM[ 1a = ehIy 310

91121b =−

= . en¼ dUc nig 3b,1a == 3111331abba == .

Page 126: HU NSEN 3 STR O N G...Problems and Solutions . GñkcUlrYmRtYtBinitübec© ... Mathematical Olympiad in China,Mathematical Olympiad Challenges, Mathematical Olympiad Treasures, International

KNitviTüaCMuvijBiPBelak

- 119 - PaK1

lMhat´TI64 eKtag r nig R erogKñaCakaMrgVg´carikkñúg nigcarikeRkA énRtIekaN ABCmYy . k¿cUrRsayfa

Rr1CcosBcosAs +co +=+

.

x¿cUrRsayfa r2R ≥

dMeNa¼Rsay k¿Rsayfa

Rr1CcosBcosAcos +=++

2BBcosAcos Ccos

2CBcos2

2Asin21Ccos 2 −++−=

++

2Csin

2Bsin

2Asin41

)2

CBcos2

CBcos(2Asin21

)2

CBcos2Asin(

2Asin21

2CBcos

2Asin2

2Asin21 2

+=

−−

+−=

−−−=

−+−=

edayeKmanbc

)cp)(bp(2Asin −−=

ab)bp)(ap(

2Bsin =

p(2Csin,

ac)cp)(a −−

=−−

Page 127: HU NSEN 3 STR O N G...Problems and Solutions . GñkcUlrYmRtYtBinitübec© ... Mathematical Olympiad in China,Mathematical Olympiad Challenges, Mathematical Olympiad Treasures, International

KNitviTüaCMuvijBiPBelak

- 120 - PaK1

eKán abc

)cp)(bp)(ap(2Csin

2Bsin

2Asin −−−

= eKTaj½

abc)cp)(bp)(ap(41CcosBcosAcos −−−

+=+

+

Rr1

R.pr.p1

R.pS1

R.SpS1

RR4

abc.p

)cp)(bp)(ap(p1

2

+=+=+

+=

×

=

−−−+=

dUcen¼ Rr

1CcosBcosAcos +=++ . fa

x¿Rsay r2R ≥

tamvismPaB :GMAM − βα≥β+α .2 eKán )bp)(ap(2)bp()ap( ≥− −−−+

)bp)(ap(2c

)cp)(ap(2bap2

−−≥

−−≥−−

eKTaj )1(21

c)bp)(ap( −≤

dUcKñaEdr )2(21

a)cp)(bp(≤

−−

Page 128: HU NSEN 3 STR O N G...Problems and Solutions . GñkcUlrYmRtYtBinitübec© ... Mathematical Olympiad in China,Mathematical Olympiad Challenges, Mathematical Olympiad Treasures, International

KNitviTüaCMuvijBiPBelak

- 121 - PaK1

nig )3(21

b)cp)(ap(≤

−− ´TMng Gg wgGg:eKán ½ KuNTMnak :n)3(,)2(,)1(

81

abc)cp)(b≤

−−

eKTaj

p)(ap( −

81

2Csin

2Bsin

2Asin ≤

eday 2C

sin2B

sin2A

sinco 41CcosBcosAs +=++ eKán

23)

81(41CcosBcosAcos =+≤++

Et Rr1CcosBcosAcos +=++

aj eKT 23

Rr1 ≤+ ¦ .

r2R ≥

Page 129: HU NSEN 3 STR O N G...Problems and Solutions . GñkcUlrYmRtYtBinitübec© ... Mathematical Olympiad in China,Mathematical Olympiad Challenges, Mathematical Olympiad Treasures, International

KNitviTüaCMuvijBiPBelak

- 122 - PaK1

lMhat´TI65 k¿cUrRsayfa x2cotxcot

x2sin1

−= x¿cUrKNna

n2

n

2aS

2a

sin

1....

sin

1

2a

sin

1asin

1++++=

dMeNa¼Rsay k¿Rsayfa x2cotxcot

x2si

tag n1

−= x2cotxcot)x(f − =

x2sin1

xcosxsin21xcos2xcos2

xcosxsin21xcos2

xsinxcos

x2sinx2cos

xs

=

in

22

2

=+−

=

−−

¼

xcos=

dUcen x2cotxcotx2sin

1−= .

x¿KNna n22

asin

Sn

2a

sin

1....

1

2a

sin

1asin

1++++=

eyIgán ∑=

=n

0kk

n )

2a

sin

1(S eday x2cotxcot

x2sin1

−=

acot

2a

cot

2a

Sn

kkn ⎟⎠⎞=

=∑ cot

2a

cot

1n

01k

−=

⎜⎝⎛ −

+

+

dUcen¼ acot2a

cotS 1nn −= + .

Page 130: HU NSEN 3 STR O N G...Problems and Solutions . GñkcUlrYmRtYtBinitübec© ... Mathematical Olympiad in China,Mathematical Olympiad Challenges, Mathematical Olympiad Treasures, International

KNitviTüaCMuvijBiPBelak

- 123 - PaK1

l atTI6Mh 6

k¿cUrRsayfa xcot2x

cot

xcos1

1 =+ KuN x¿KNnapl

)

2a

cos

11).....(

2a

cos

11)(

2a

cos

11)(

acos1

1(Pn +=

n

+++ 2

dMeNa¼Rsay

k¿Rsayfa xcot2x

cot

xcos1

1 =+ eyIgtag

xcos1

1)x(A +=

xcot2x

cotxtan

2xco

== tanxcosxsin

.

2x

sin

2x

s

2x

sinxcos

xsin2x

cos

2x

sinxcos

2x

sin2x

cos2

xcos2x

cos2

x1x

22

=

===+

=

coscos

dUcen¼ xcot2x

cot

xcos1

1 =+ . x¿KNnaplKuN )

2a

cos

11).....(

22

acos

11)(

2a

cos

11)(

acos1

1(P

n

n ++++=

acot.2

atanP = 1nn + .

Page 131: HU NSEN 3 STR O N G...Problems and Solutions . GñkcUlrYmRtYtBinitübec© ... Mathematical Olympiad in China,Mathematical Olympiad Challenges, Mathematical Olympiad Treasures, International

KNitviTüaCMuvijBiPBelak

- 124 - PaK1

lMhat´TI67 eK[sIVúténcMnYnBit n kMnt´elI eday ½ )U( IN

22

U0 = nig INn,211− U

U2

n1n ∈∀

−=+

aGnuKmn_én . KNna nU C

n

dMeNa¼Rsay KNna GnuKmn_én ½

nU Ca n

eyIgman4π

sin22

0 == U

sin2

sin11

2U11

U1

−=

22

0 =−−

=−

«bmafavaBitdltYTI p KW 2pp s=2π

inU + ayfavaBitdl YTI eyIgnwgRs ´t )1p( + KW 3p1p 2

πsinU ++ = Bit

an eyIgm2

U11U

2p

1p

−−=+ Ettamkar«bma 2pp 2

πsinU +=

eyIgán 2

sin11U 1p+

−−= 2

π2p

2+

3p22 +

dUcen¼

32p sin22π

cos1 +− p2

πsin2

π

2

+=== Bit

2nn 2π

sinU += .

Page 132: HU NSEN 3 STR O N G...Problems and Solutions . GñkcUlrYmRtYtBinitübec© ... Mathematical Olympiad in China,Mathematical Olympiad Challenges, Mathematical Olympiad Treasures, International

KNitviTüaCMuvijBiPBelak

- 125 - PaK1

lMhat´TI68 cUrKNna

9π7

cos9π4

cos9π

cosS 333 +−=

dMeNa¼Rsay KNna

9π7

cos9π4

cos9π

cS = os 333 +− tamrUbmnþ acos3acos4a3cos 3 −= ¦ a3cos

41

acos43

acos3 += GacsresrCa ½ kenßamEdl[

)3π7

cos3π4

s(cos43

S = co3π

(cos41

)9π7

cos9π4

cos9π

+−++− tag

9π7

cos9π4

cos9π

cosM = +− y eda

9π13

cos9π4

cos =−

9π13

cos9π7

cos9π

cosM ++= KuNnwg 3π

sin2 eKán

0)9π7

cos(πsi23

M2 n29π16

sin9π2

sin

9π10

sin9π16

sin9π4

9π10

sin)9π2

sin(9π4

sin3

sin9π13

cos23π

sin9π7

cos23π

sin9π

cos23π

sinM2

=−=+=

−+−+−−=

++=

eKTaján

sin2

M.2

0M =

Page 133: HU NSEN 3 STR O N G...Problems and Solutions . GñkcUlrYmRtYtBinitübec© ... Mathematical Olympiad in China,Mathematical Olympiad Challenges, Mathematical Olympiad Treasures, International

KNitviTüaCMuvijBiPBelak

- 126 - PaK1

tag 23

21

21

21

3π7

cos3π4

cos3π

cosN =++=+−= eKán

83

N41

M43

S =+= .

dUcen¼ 83

97cos

94cos

9cosS 333 =

π+

π−

π= .

lMhat´TI69 eK[RtIekaN mYymanmMukñúgCamMuRsYc .

ayfa Btan.AtanCtanBtanAtan

ABC

k¿cUrRs tan. C=++ ¿TajbBa¢ak´fa x 33CtanBtanAtan ≥++ .

dMeNa¼Rsay k¿Rsayfa tan Ctan.Btan.AtanCtanBtanA =++

= ¦ eyIgman BA ++ πC CπBA −=+ án eK )Cπtan()BAtan( −=+

CtanBtanAtanCtanBtanAtan

CtanBtanBtan

Atan1Atan

+−=

−=

tan.AtanBtanA

+−

+

dUcen¼ .BCtantan Ctan=++ .

Page 134: HU NSEN 3 STR O N G...Problems and Solutions . GñkcUlrYmRtYtBinitübec© ... Mathematical Olympiad in China,Mathematical Olympiad Challenges, Mathematical Olympiad Treasures, International

KNitviTüaCMuvijBiPBelak

- 127 - PaK1

x¿TajbBa¢ak´fa 33CtanBtanAtan ≥++ CamMuRsYc ( tamsm μtikm μ )

eKTaj PaBkUsIueyIgGacsresr ½

eday C,B,A

0Ctan,0Btan,0Atan >>>

tamvism3 Ctan.Btan.Atan3CtanAtan ≥+

eday tan.Btan.AtanCtanBan

Btan + tAtan C=++

eKán 3 CtanBtat nAtan3BtanBtanAan ++≥++

2 ≥++

+ (tan27)CtanBtanA(tan 3

27)CtanBtanA(tan

)CtanBtanA +≥++ dUcen¼ 33CtanBtanAtan ≥++ .

Page 135: HU NSEN 3 STR O N G...Problems and Solutions . GñkcUlrYmRtYtBinitübec© ... Mathematical Olympiad in China,Mathematical Olympiad Challenges, Mathematical Olympiad Treasures, International

KNitviTüaCMuvijBiPBelak

- 128 - PaK1

lMhat´TI70 KNnaplbUk ½ n 999....999....999999S ++++= manelx cMnYn elx ) . ( 9 n

dMeNa¼Rsay KNnaplbUk ½

9.. 99......999. .999999Sn

)9n( elx++++=

9)10n9(10

n9

110.10n

−−

=

)1...111()10...10)110(....)110()110()110(

1n

n32

n32

+−=

++++−+++

−++−+−+−=

+

dUcen¼

1010( +=

9)10n9(10S

2n

n+−

=+

.

Page 136: HU NSEN 3 STR O N G...Problems and Solutions . GñkcUlrYmRtYtBinitübec© ... Mathematical Olympiad in China,Mathematical Olympiad Challenges, Mathematical Olympiad Treasures, International

KNitviTüaCMuvijBiPBelak

- 129 - PaK1

lMhat´TI71 cUrkM ntRKb´KUtémøKt´viC¢man ebIeKdwgfacMnYn ½

)b,a(

1bab2a

32

2

+− CacMnYnKt´viC¢manEdr .

dMeNa¼Rsay nt´RKb´KUtémøKt´viC¢man ½ k

kM )b,a(

y k1bab2

a32

2

=+−

Edl *INk∈ 32eKán 2a2 )1(0)1b(kakb =−+

42

DIsRKImINg´énsmIkar )1b(k4 3bk4 −−=Δ 22 k4)bkb2( 2b−+−=Δ mIkar mancemøIykñúg lu¼RtaEt s )1( *IN Δ CakaerRákd

mann&yfa 2222 dbk4)bkb2( =−+−=Δ

d

Edl CacMnYnKt´. -ebI 0bk4 2 =− ¦

4 bk

2

=

eyIgTTYlán 2

bbb2

kb2a3

2 −=−= ¦

2ba =

Page 137: HU NSEN 3 STR O N G...Problems and Solutions . GñkcUlrYmRtYtBinitübec© ... Mathematical Olympiad in China,Mathematical Olympiad Challenges, Mathematical Olympiad Treasures, International

KNitviTüaCMuvijBiPBelak

- 130 - PaK1

eday a b, CacMnYnKt´viC¢man ehtuen¼eKRtUv[ ½ INp,p2b ∈∀=

eKTaj pp8p4

ehIy

)p2( 42

−=− )p2(2a 2=

pp2== .

2a

2,p(;)p2,pp4dUcen¼ 8()b, *INp,)pa( ∈∀−=

-ebI 0b

k4 2 >−

eKán 4)2 bkb2( 22222 *INk,)1bkb2(dbk ∈∀+−≥=−+− 2¦ )1b(k4 2 0)1b( ≤−+− án eKTaj

IsmI1b =

kñúgkrN kar )1( køayCa 0ak2a2 =− naM[ k2a = dUcen¼ )1,k2()b,a( = cMeBa¼RKb´ *INk∈ . -ebI 0bk4 − 2 <

)bkb −eKán k42( 222222 )1bkb2(db −<=−+− )bkb2 2222 <smmUl 4( 2 0)1bkb2(bk −−−−+−

¦ k4(b2 0)1k4()1b(b2)3 <−+−+− ( minBitkñúg ) srubmkeKánKUcemøIybImanragdUcxageRkam ½

*IN

)k2,kk8(;)k2,k(;)1,k2()b,a( 4 −= *INk∈

Page 138: HU NSEN 3 STR O N G...Problems and Solutions . GñkcUlrYmRtYtBinitübec© ... Mathematical Olympiad in China,Mathematical Olympiad Challenges, Mathematical Olympiad Treasures, International

KNitviTüaCMuvijBiPBelak

- 131 - PaK1

´TIlMhat 72 cUrbgHajfa ½ 1

ab8ca 22

cca8bbc8 2

≥ba

++

++

+

RKb´cMnYnBit ¢man . cMeBa¼ viC c,b,a

dMeNa¼Rsay

ajfa bgH 1ab8

c≥

cca8bbc8aba

222 ++

++

+

CadMbUgeyIgRtUvRsayfa

34

34

34

34

aa2

cbabc8a ++≥

+

mmUl

vismPaBen¼s )bc8a(a)cba( 234

234

34

34

+≥++ smmUl bca8cb2ca2ba2cb 3 3

434

34

34

34

34

34

38

≥8

+++ amvismPaB eKman

+

t GMAM − 34

34

38

38

cb2cb ≥+ eKTaj bca8cb2ba2cb 3333 ca2 3

434

34

34

344488

≥++++ Bit

tuen¼ eh ( )1cba

abc8a

a

34

34

34

34

2

++≥

+

Page 139: HU NSEN 3 STR O N G...Problems and Solutions . GñkcUlrYmRtYtBinitübec© ... Mathematical Olympiad in China,Mathematical Olympiad Challenges, Mathematical Olympiad Treasures, International

KNitviTüaCMuvijBiPBelak

- 132 - PaK1

dUcKñaEdreKTaj ( )2cba

bac8b

b

34

34

34

34

2

++≥

+

ehIy ( )3cba

cab8c2

c

34

34

34

34

++≥

edaybUkvismPaB Gg:nigGg:eKán +

)3(,)2(,)1(

1ab8c

cca8b

bab8a c 222

≥+

++

. ++

lMhat´TI73 eK[ a( n )CasIVúténcMnYnBitEdl

21a1 =

b´cMnYnKt´viC¢man eyIgman nigcMeBa¼RK n1aa

aan

2n

2n

1 =+n +−

cUrRsaybBa¢ak´facMeBa¼RKb´cMnYnKtviC¢man n eyIgman ½ n3 aa 21 1a.........a <+++ .

dMeNa¼+

Rsay RsaybBa¢ak´fa 1a.........aaa n321 <++++ eyIgman

1aaaa 1n =+

n2

n

2n

+−

Page 140: HU NSEN 3 STR O N G...Problems and Solutions . GñkcUlrYmRtYtBinitübec© ... Mathematical Olympiad in China,Mathematical Olympiad Challenges, Mathematical Olympiad Treasures, International

KNitviTüaCMuvijBiPBelak

- 133 - PaK1

tag n

n a1b = ena¼TMnak´TMngEdl[køayeTACa ½

n2 +−=

bb1 −=−

1bbb n1n+

eKTaj 1nb + n2

n

nnnn1n 1b)b(b1b−

−=

−=

nn1 )1b(b1n

b11

111

b −=−

+

+

eKTaj 1b1bb

a1nnn

n −−

−==

+

111

n21n21 .aa ++

b1......

b1

b1a... +++=+

11b

111b1b 1n1 − +

111b1b 121 −−− +

)1b

11b

1(...)1b

11b

1()

1n

nn32

<−

−=−

−=

−−

++−

−−

+

+

BIeRBa¼

11( −=

2a1b

1

dUcen¼ 1a.........aa

1 == .

a n321 <++++ .

Page 141: HU NSEN 3 STR O N G...Problems and Solutions . GñkcUlrYmRtYtBinitübec© ... Mathematical Olympiad in China,Mathematical Olympiad Challenges, Mathematical Olympiad Treasures, International

KNitviTüaCMuvijBiPBelak

- 134 - PaK1

lMhat´TI74 cUrkMntRKb´KUtémøKt´ 3n,m ≥ ebIeKdwgfacMeBa¼RKb´ cMnYnKtviC¢man eKman a

1aa1aa

2n

m

−+−+ CacMnYnKt´ .

dMeNa¼Rsay

´KUtémøKt´viC ½ kMnt´RKb ¢man ( )n,m

edIm,I[ 1aa

an1aa2n

m

−+−+ CacMnYnKt´lu¼RtaEt 2 −1a+

aktþarYmén C 1aam −+ ehIy . Igyk

nm >

ey *INk,knm ∈+= Igán ½

k1k2nk

knm

−+−+−+

−+=−++

+

,I[

ey

)1aa)(a1()1aa(a1aa1aa

=

tamTMnak´TMngen¼edIm 1aa 2n −+ CaktþarYmén a 1am −+

lu¼RtaEt 1kn += n

ig 2k = . .

dUcen¼ )3,5()n,m( =

Page 142: HU NSEN 3 STR O N G...Problems and Solutions . GñkcUlrYmRtYtBinitübec© ... Mathematical Olympiad in China,Mathematical Olympiad Challenges, Mathematical Olympiad Treasures, International

KNitviTüaCMuvijBiPBelak

- 135 - PaK1

lMhat´TI75 eK[bIcMnYnKt c,b,a Edl ba´viC¢man = 10c++ . cUrrktémøFMbMputén cbaP ××= . dMeNa¼Rsay rktémøFMbMputén cbaP ××=

aM[ ceyIgman 10cba =++ n ba01 −−= )b10(b10(ab 2 −eyIgán baP aa)b +−=−−=

2tag )ba(P ab10() 2ba −+−= eKán 2bb10ab2)a('P −+−= bI eKTaján e 0)a('P =

2b5a −=

eday a(''P *INb,0b2) ∈∀<−= øGtibrma mø ehtuen¼ )a(P mantém cMeBa¼té

2b5a −= .

a¼ RtUvEt YnKU I

eday *INa∈ en CacMnb

eb k2b = ena¼ k5a −= ehIy k5k2)k5(10c −=−−− =

Page 143: HU NSEN 3 STR O N G...Problems and Solutions . GñkcUlrYmRtYtBinitübec© ... Mathematical Olympiad in China,Mathematical Olympiad Challenges, Mathematical Olympiad Treasures, International

KNitviTüaCMuvijBiPBelak

- 136 - PaK1

eday ,a ena¼ ⎪⎩

⎨*INc,b ∈ ⎪⎧

≥−=≥=≥−=

1k5c1k2b

1k5a ¦ 4k1 ≤≤

= n

-cMeBa¼ k 1

eKá 4c,2b,4a === naM[ 32424P =××=

n 3a

-cMeBa¼ 2k =

eKá c,4b, 3== P= naM[ =× 36343×= -cMeBa¼ 3k = eKán 2c,6b,2a == naM[ 242P= 26 =××= -cMeBa¼ 4k =

n a eKá 1b,1 c,8 === naM[ 32181P =××= témøGtibrmaén dUcen¼ c.b.aP = es wg .

μIn 36

Page 144: HU NSEN 3 STR O N G...Problems and Solutions . GñkcUlrYmRtYtBinitübec© ... Mathematical Olympiad in China,Mathematical Olympiad Challenges, Mathematical Olympiad Treasures, International

KNitviTüaCMuvijBiPBelak

- 137 - PaK1

lMhat´TI76 eKmansmPaB

ba1

bxcos

axin 44s

+=+

0Edl b,a 0ba,0 ≠+≠≠ a cUrbgHajf 33

8

3

8

)bxcos

axsin

=+ .

eNa¼

ba(1+

dM Rsay gHajfa b 33

8

3

8

)ba(1

bxcos

axsin

+=+

eKman ba

1b

xcosa

xsin 44

+=+

M[ na abxcos)ba(axsin)ba(b 44 =+++

0)xsinbxcosa(

0xsinbxsinxcosab2xcosa)xcosx(sinabxcosabxcosaxsinbxsinab

222

422242

422442424

=−

=+−

+=+++

eKTaj ba

1ba

xa

xsin=

cosxsinb

xcos 2222

+++

==

naM[ 44

8

4

8

)ba(1

bxcos

axsin

+==

Page 145: HU NSEN 3 STR O N G...Problems and Solutions . GñkcUlrYmRtYtBinitübec© ... Mathematical Olympiad in China,Mathematical Olympiad Challenges, Mathematical Olympiad Treasures, International

KNitviTüaCMuvijBiPBelak

- 138 - PaK1

eKTaj )1()ba(

aa

xsin43

8

+= nig )2(

)ba(b

bxcos

43

8

+=

nig Gg:nigGg: eKán ½ bUksmIkar )1( )2(

343

8

3

8 cxsin+

)ba(1

)ba(ba

bxos

a +=

++

=

dUcen¼ 33

8

3

8

)ba(1

bxcos

axsin

+=+ .

lMhat´TI77 eK[RtIekaN ABC mYYy . bgHajfaebI

3Ctan,

3Btan,

3Atan

Ikar bxaxx:)E( 23Ca¦ssm c 0=+++ n ena¼eKá cb3a3 +=+ .

dMeNa¼Rsay karbgHaj ½ eyIgman π=++ CBA

MukñúgRtIekaN ) yIgán ( plbUkm ABC

e ]3C)

3B

3Atan[()

3C

3B

3Atan( ++=++

Page 146: HU NSEN 3 STR O N G...Problems and Solutions . GñkcUlrYmRtYtBinitübec© ... Mathematical Olympiad in China,Mathematical Olympiad Challenges, Mathematical Olympiad Treasures, International

KNitviTüaCMuvijBiPBelak

- 139 - PaK1

)1()

3Ctan

3Btan

3anta

3tan

3(tan1 +−

Ct3

nBA3Ctan

3Btan

3AtanCta

3Btan

3

3Ctan.

3Btan

3Atan

3Btan

3Atan

3Ctan

3Btan

3A

3BtanA

3Ctan

31

3Ctan)

3B

3Atan(

+

+

+

++

++=

A3

ntan3

−=

A+

11

−−

tan1

3

tan

3tan =

π

)3BAtan(

)3

tan(+

CBA ++

eday 3Ctan,

3Btan,

3Atan Ca¦srbs´smIkar

ena¼tamRTwsþIbTEvüteKmanTMnak´TMng ½ )E(

)2(a3Ctan

3Btan

3Atan −=++

)4(c2Ctan.

2Btan.

2Atan

)3(b2Ctan

2Atan

2Ctan

2Btan

2Btan

2Atan

−=

=++

ykTMnak´TMng nig CYskñúgsmIkar eKán ½ )3(,)2( )4( )1(

Page 147: HU NSEN 3 STR O N G...Problems and Solutions . GñkcUlrYmRtYtBinitübec© ... Mathematical Olympiad in China,Mathematical Olympiad Challenges, Mathematical Olympiad Treasures, International

KNitviTüaCMuvijBiPBelak

- 140 - PaK1

b1ca3

−+−

= ¦ cab33 +−=− dUcen¼ cb3a3 +=+ .

lMhat´TI78 k¿ cUrKNnatémø®ákdén

10sin π nig

10cos π

x¿ cUrRsayfa 10

sin)yx(4)yx(x 22222 π+≤−+

RKb´cMnYn IRy,x ∈ .

dMeNa¼Rsay k¿ KNnatémø®ákdén

10sin π nig

10cos π

eKman 1032

210π

−= eKán

ππ

103cos)

103

2sin(

102sin π

−π

tamrUbmnþRtIekaNmaRt ½ nig acosasin2a2sin = acos3acos4a3cos 3 −=

Page 148: HU NSEN 3 STR O N G...Problems and Solutions . GñkcUlrYmRtYtBinitübec© ... Mathematical Olympiad in China,Mathematical Olympiad Challenges, Mathematical Olympiad Treasures, International

KNitviTüaCMuvijBiPBelak

- 141 - PaK1

)1010

sin1(43sin2

10cos43

10sin2

10cos4

10cos

10cos

10sin2

103cosco

10sin2

2

3

π−−=

π

π−=

π

π−

π=

ππ 3

10s

2

π=

ππ

¦ 0sin2sin4 2 =11010

−π

−π tag 0

10 eKán t4 2

sint >π

>

=

0541',01t2 =+=Δ=−− ¦s eKTaj 0

451−t1 <= ( minyk )

451t, 2

+=

dUcen¼ 4

5110

sin +=

π . eday 1

10cossin 22

10=

π+

π

naM[ 4

5210)4

51(110

2 −=

+−=

π cos

dUcen¼ 410

cos =5210 −π .

sayfa x¿ R10

sin)yx(4)2 ≤yx(x 2222 π+−+

tagGnuKmn_ 10

sin)yx(4)yx(x)y;x(f 22222 π+−−+=

Page 149: HU NSEN 3 STR O N G...Problems and Solutions . GñkcUlrYmRtYtBinitübec© ... Mathematical Olympiad in China,Mathematical Olympiad Challenges, Mathematical Olympiad Treasures, International

KNitviTüaCMuvijBiPBelak

- 142 - PaK1

eKán ½ 222222 )

451)(yx(4yxy2xx)y;x(f +

+−+−+=

IRy,x,0y2

15x2

15

y2

15xy2x2

15

y5 2

21xy2x

251

253)yx(yxy2x2

1626)yx(4yxy2x2

22

2

2222

2222

∈∀≤⎟⎟⎠

⎜⎜⎝

−−

⎟⎟⎠

⎞⎜⎜⎛ +

++−

+−−

−=

++−+−=

++−+−

5=

2⎞⎛ +

+=

=

dUcen¼ 10

sin)yx(4)yx(x 22222 π+≤−+ .

Page 150: HU NSEN 3 STR O N G...Problems and Solutions . GñkcUlrYmRtYtBinitübec© ... Mathematical Olympiad in China,Mathematical Olympiad Challenges, Mathematical Olympiad Treasures, International

KNitviTüaCMuvijBiPBelak

- 143 - PaK1

lMhat´TI79 eK[sIVúténcMnYnBit kMnt´eday)U( n 4

nsin.2Un

= Edl . k-cUrbgðajfa

*INn∈

4nsin

4ncos

4)1n(cos.2 π

−π

=π+

x-Taj[ánfa 4

)1n(cos)2(4

ncos)2(U 1nnn

π+−

π= +

K-KNnaplbUk ½ n321n U.........UUUS ++++= CaGnuKmn_én .

dMeNa¼

n

Rsay a k-cUrbgHajf

4nsin

4ncos

4)1n(cos.2 π

−π

=π+

tamrUbmnþ bsinasinbcosacos)bacos( −=+ Igán ½ ey

)4

sin4

nsini4

cos4

n(cos2

)44

ncos(24

)1n(cos2

ππ−

ππ=

π+

π=

π+

4

nsin4

ncos)4

nsin22

4ncos

22(2

4)1n(cos2 π

−π

−π

=π+

Page 151: HU NSEN 3 STR O N G...Problems and Solutions . GñkcUlrYmRtYtBinitübec© ... Mathematical Olympiad in China,Mathematical Olympiad Challenges, Mathematical Olympiad Treasures, International

KNitviTüaCMuvijBiPBelak

- 144 - PaK1

dUcen¼ 4

nsin4

ncos4

)1n(cos.2 π−

π=

π+ .

x-Taj[ánfa 4

)1n(cos)2(4

( 1nn

ncos)2U n π+−

π= +

an eyIgm4

nsin4

ncos4

)1n(osc.2 π−

π=

π+ naM[

4)1n(cos2

4ncos

4nsin π+

−π

KuNGg:TaMgBIrnwg n)2( eKán

4)1n(cos)2(

4ncos)2(

41nnnsin)2( n π+

−π

= +

Ucen¼

π

d4

)1n( π+cos)2(4

ncos)2(U 1nn −nπ

= + . K-KNnaplbUk n321n U.........UUUS ++++=

eyIgán

4

)1n(cos)2(4

cos2

4)1 πk(cos)2(

4kcos)2(

1n

n

1k

1kk

π+−

π=

⎥⎦⎤

⎢⎣⎡ +

−Snπ

=

+

=

+∑

dUcen¼ 4

)1n(cos)2(1S 1nn

π+−= + .

Page 152: HU NSEN 3 STR O N G...Problems and Solutions . GñkcUlrYmRtYtBinitübec© ... Mathematical Olympiad in China,Mathematical Olympiad Challenges, Mathematical Olympiad Treasures, International

KNitviTüaCMuvijBiPBelak

- 145 - PaK1

lMhat´TI80 eK[ n cMnYnBit 0a.......;;a;a;a n321 ≥ . eKtag *NSn In,

na.....aaa n321 ∈∀

++++=

yfaebI

cUrRsa 1n1nan3211n a.......a.a.aS +

++ ≥ ena¼eKán n

3n .....S ≥

n21 aaaa .

dMeNa¼Rsay ebI Rsayfa 1n

1nn3211n aa.......a.a.aS +++ ≥

ena¼eKán nn321n

eyIgman a.....aaaS ≥

1n1nn3211n aa.......a.a.aS ≥ +

++ eday

na.....aaa

S n321n

++++ =

ena¼ 1n

aa....aaaS 1n

1nn321

++++++

= + eKán ½

+

( )1aa.........aaa 21 aaa

1naa..... 1n

1nn3211nn3 +

++ ≥

++++

k

++

yn

a.....aaaa n321

1n

++++=+ CYskñúg ( )1 eKán ½

Page 153: HU NSEN 3 STR O N G...Problems and Solutions . GñkcUlrYmRtYtBinitübec© ... Mathematical Olympiad in China,Mathematical Olympiad Challenges, Mathematical Olympiad Treasures, International

KNitviTüaCMuvijBiPBelak

- 146 - PaK1

nn21

n21

n21 a.a≥n

n2

n21n21

1nn21

1n n221

n2

1n n21n21

n21n

a.........a.an

a.......aa

a......n

a.......a

na...aa

a.....a.an

a.........aan

a....a....a.a

na.......a

na....aa

.a...a.ana.....aa

≥+++

⎟⎠⎞

⎜⎝

+++

++≥⎟

⎠⎞

⎜⎛ +++

+++≥

++

+++≥

1a⎛

1n

1 a.a

a1n+

+

21 a....aa

...+

++++

+

+

+

+++

eKTaj nn321n

dUcen¼ ebI a.....aaa S ≥

1n1nn3211n aa.......a.a.aS +

++ ≥ ena¼eKán n

n321n a.....aaaS ≥ .

lMhat´TI81 cUrbgHajfa

101

10099......

65.

43.

21

151

<<

dMeNa¼Rsay bgHajfa

101

10099......

65.

43.

21

151

<< an eyIgm )1n(n)1n( +>+ 2n1n2:*INn +=+∈∀

eKTaj 1n

n2n21n2

+>

++

Page 154: HU NSEN 3 STR O N G...Problems and Solutions . GñkcUlrYmRtYtBinitübec© ... Mathematical Olympiad in China,Mathematical Olympiad Challenges, Mathematical Olympiad Treasures, International

KNitviTüaCMuvijBiPBelak

- 147 - PaK1

eKán

251

5049......

32.

21

10099......

65.

43

=>

KuNnwg 21 eKán

2101

10099........

65.

43.

21

>

eday 151

2101

>

eKTaj )1(151

10099....

65.

43.

21

> müa¨geTot

)1n2)(1:*INn −∈∀ n2(2)1n2()1n2(n4 +>++−= eKTaj

1n21n2

1n2n2

−+

>−

rW 1n21n2

n2n2 − 1

+−

<

eKán 1011

10199....

75.

53.

31

10099.....

65.

43.

21

=<

eday 101

1001

1011

=<

eKTaj )2(101

10099....

65.

43.

21

< Mng nwg eKán ½tamTMnak´T )1( )2(

101

10099......

65.

43.

21

151< < .

Page 155: HU NSEN 3 STR O N G...Problems and Solutions . GñkcUlrYmRtYtBinitübec© ... Mathematical Olympiad in China,Mathematical Olympiad Challenges, Mathematical Olympiad Treasures, International

KNitviTüaCMuvijBiPBelak

- 148 - PaK1

lMhat´TI82 eK[ n2 ....; cMnYnBit ;b;b;a.....;;a;a .

a....b 2n

22

21

2n

22

21

2n22 ++++++≤++

n21n21 b

cUrRsayfa ½ )b.....bb)(a....aa()baba( n11 +

dMeNa¼Rsay

.....bb)(a....aa()ba....baba( 222

21

2n

22

21

2nn2211 +++++≤+++

eyIgeRCIserIsGnuKmn_mYykMnt´

Rsayfa ½ )bn+

IRx∈∀ eday ½ )bx..)bxa()b

2n

221nn2211

22n

22

2

2n

222

21

+++++++++

+++++

eday )b...bb(x)ba..baba(2x)a...aa()x(f 21 +=

a(...xa()x(f n1 +=

IRx∈∀ RtIFa 0)x(f ≥ Canic©ena¼

eday KánCanic©

⎩⎨⎧

≤Δ>

0'0af

0a.....aaa 2n

22

21f >+++=

ehtuen¼e 0'≤Δ . ( ) 0)b....bb)(a....aa(ba. ...baba' 2

22

1n211 ++++++=Δ .... 2n

22

21

2n

2n2 ≤+++−

dUcen¼ a()ba....baba( 2n

22

21

2n

2nn2211 ++++≤+++ )b.....bb)(a....a 2

22

1 ++

Page 156: HU NSEN 3 STR O N G...Problems and Solutions . GñkcUlrYmRtYtBinitübec© ... Mathematical Olympiad in China,Mathematical Olympiad Challenges, Mathematical Olympiad Treasures, International

KNitviTüaCMuvijBiPBelak

- 149 - PaK1

lMhat´TI83 eK[ )( nig y( nxn CasIVútcMnYnBitkMnt´elI

0 = nigTMnak´TMngkMenIn ½ n1n yx3xx +=+ nig RKb´

cUrKNna nig CaGnuKmn_én .

dMeNa¼

) IN

eday ,5x0 =3

1y2

nn3

nn2

n1n yyx3y +=+ INn∈

n

x ny n

Rsay

eKman nnn1n +=+ nig

Ikar eKá)yx(y

n

3nn1n

+=+

+=+

ln{ n

KNna nx nig ny CaGnuKmn_én23

n

)1(yx3xx

)2(yyx3y 3nn

2n1n +=+

bUksm nig 2( n ½ )1( )

)yxln(.3)yxln(x 1n ++

n1n1n ++

TMnak´TMngen¼bBa¢ak´fa x( n })y+ CasIVútFrNImaRtman ersug nigtYdMbUg 3q = 6ln)yxln( 00 =+ .

eKán 6ln3)yxln( nnn =+

Page 157: HU NSEN 3 STR O N G...Problems and Solutions . GñkcUlrYmRtYtBinitübec© ... Mathematical Olympiad in China,Mathematical Olympiad Challenges, Mathematical Olympiad Treasures, International

KNitviTüaCMuvijBiPBelak

- 150 - PaK1

eKTaj nig eKán ½

.3yy 3

n

−+

¢ak´fa

)3(6yxn3

nn =+

dkksmIkar yx =−

)1( )2(

)yx()xln()x(

nn1nn

nn11n

−=++

+ ln1

TMnak´TMngen¼bBa })yxln({ nn − CasIVútFrNImaRtman ersug nigtYd 003q = MbUg ln( 4ln)yx =− .

Taj nig eKán

eKán 4ln3)yxln( nn− n =

eK )4(4yxn3

nn =−

bUksmIkar 3( ) )4(nn 33

n 46x2 +=

eKTaj 2

46xnn 33

= . n+

ar nig eKán −= dksmIk y2)3( )4(nn 33

n 46

eKTaj 2

yn46

nn 33 −= .

dUcen¼ 2

46xnn 33 +

n = nig 2

46ynn 33

n−

= .

( anPaKTI2 sUmG )

sUmrgcaMG rKuN

Page 158: HU NSEN 3 STR O N G...Problems and Solutions . GñkcUlrYmRtYtBinitübec© ... Mathematical Olympiad in China,Mathematical Olympiad Challenges, Mathematical Olympiad Treasures, International

KNitviTüaCMuvijBiPBelak

- 151 - PaK1

Éksareyag ( Referent )

Hu hematical Chal es dward J. Bar beau, M ray S. Klamkin)

plex Numbers Fro A to … Z itu Andresscu , Dorin Andrica )

a Oly in China at ly hallenges

itu Andresscu & n Gelca) ematical Oly d Treasures

ndreescu , Bogdan enescu ) 7. International Mathematical Olympiads 1959-1977

8. The IMO Compendium ( 1959-2004 )

(By Titu Andreescu , Dorin Andrica )

1. 103 Trigonometry Problems (From the training of the USA IMO Team)

2. Five ndred Mat leng (By E ur

3. Com m (By T

4. Mathem tical mpiad 5. Mathem ical O mpiad C (By T Razva

6. Math mpia (By Titu A

( Samuel L. Greitzer )

9. 360 Problems for Mathematical Contest

Page 159: HU NSEN 3 STR O N G...Problems and Solutions . GñkcUlrYmRtYtBinitübec© ... Mathematical Olympiad in China,Mathematical Olympiad Challenges, Mathematical Olympiad Treasures, International

KNitviTüaCMuvijBiPBelak

- 152 - PaK1